Download as pdf or txt
Download as pdf or txt
You are on page 1of 52

Team PRTC CPA Review

Final Pre-board Examination for October 2022 Batch


Personal copy of Barbin, Rafael R. (rafaelbarbin123@gmail.com)
Global Reciprocal Colleges,Inc-Caloocan

SUGGESTED ANSWERS & SOLUTION TEAM PRTC


FINAL PRE-BOARD EXAMINATION September 30, October 1 and 2, 2022

MANAGEMENT SERVICES

1. C* 21. D 41. B 61. C


2. B 22. D 42. A 62. A
3. D* 23. B 43. C 63. D
4. D 24. B* 44. B* 64. B
5. D 25. B 45. C* 65. C
6. D* 26. C* 46. D 66. D
7. C 27. D 47. B 67. D*
8. C 28. A 48. D 68. A
9. D 29. C 49. C 69. B
10. C 30. D 50. C 70. B
11. D 31. A* 51. C
12. A 32. B 52. B
13. D 33. C 53. B
14. C 34. A* 54. D
15. B 35. A* 55. B
16. A 36. A 56. A
17. A* 37. C* 57. C
18. D 38. C 58. C
19. D 39. B 59. A
20. A 40. C 60. A*

*Note: Items with asterisk pertain to questions where the common answer of the examinees is different from the
correct answer indicating topics where examinees either find difficult, confusing, or are not familiar with. Review or
revisit those topics in your final preparation for the LECPA.

Exam insights:

Highest Score: 94%


Average Score: 47%

Reviewer’s Feedback:

Some of the items noted in asterisk above are difficult because these are multidisciplinary problems requiring
application of concepts across different topics in management services. Some problems, while tricky, are answerable if
the examinees have strong conceptual foundation.

Page 1 of 52
Personal copy of Barbin, Rafael R. (rafaelbarbin123@gmail.com)
Global Reciprocal Colleges,Inc-Caloocan

AUDITING

1. C 21. A 41. A 61. B


2. D 22. C* 42. B 62. A
3. D 23. C 43. B* 63. A*
4. D 24. D 44. A 64. D
5. D 25. B 45. C 65. A
6. D 26. C 46. C 66. C
7. B 27. B 47. C 67. D
8. A 28. D 48. B 68. B
9. D 29. D* 49. B 69. C
10. B 30. C 50. B 70. D
11. A 31. C 51. C
12. A 32. D 52. B
13. B 33. C* 53. A
14. A 34. D 54. B
15. A 35. A 55. B
16. C 36. A* 56. B
17. B 37. D 57. D
18. D* 38. B 58. D
19. D 39. C 59. C
20. C 40. C* 60. B

*Note: Items with asterisk pertain to questions where the common answer of the examinees is different from the
correct answer indicating topics where examinees either find difficult, confusing, or are not familiar with. Review or
revisit those topics in your final preparation for the LECPA.

Exam insights:

Highest Score: 90%


Average Score: 51%

Reviewer’s Feedback:

Examinees have difficulty answering questions related to audit procedures applied to specific accounts as well as audit
reporting. For auditing practice, most mistakes were due to carelessness in applying accounting knowledge. Because
accounting standards provide the criteria whether the financial statements of our clients are fairly stated, auditors
must thoroughly understand those accounting standards on top of auditing knowledge.

Page 2 of 52
Personal copy of Barbin, Rafael R. (rafaelbarbin123@gmail.com)
Global Reciprocal Colleges,Inc-Caloocan

TAXATION

1. C 21. D 41. D 61. B*


2. B 22. D 42. C 62. D
3. B 23. A 43. C* 63. C
4. A* 24. C 44. D 64. B
5. D 25. C 45. D 65. B
6. D 26. D* 46. C 66. B*
7. D 27. A 47. C* 67. D*
8. A 28. C 48. C* 68. C*
9. D* 29. A/C 49. C 69. C
10. C* 30. D 50. A 70. B
11. A* 31. D 51. A*
12. C 32. D 52. A*
13. A 33. C 53. A*
14. C 34. A 54. B
15. A 35. D 55. D*
16. A 36. C* 56. D*
17. A* 37. C 57. B
18. C 38. D 58. C
19. D 39. D 59. C
20. C 40. C 60. C

*Note: Items with asterisk pertain to questions where the common answer of the examinees is different from the
correct answer indicating topics where examinees either find difficult, confusing, or are not familiar with. Review or
revisit those topics in your final preparation for the LECPA.

Exam insights:

Highest Score: 89%


Average Score: 44%

Reviewer’s Feedback:

Most questions in the preboards are applications of the rules. Questions are relatively challenging but not entirely
difficult. Once you see the answers, you'll know how easy it was. Learn from it. Be better next time. Remember that
you are better than what the scores tells you. The test is just a test of how deep your understanding about taxation is.
You know a lot more than you think.

Page 3 of 52
Personal copy of Barbin, Rafael R. (rafaelbarbin123@gmail.com)
Global Reciprocal Colleges,Inc-Caloocan

REGULATORY FRAMEWORK FOR BUSINESS TRANSACTIONS

1. B* 21. A 41. D* 61. B 81. A


2. E 22. B* 42. A 62. C 82. D
3. C 23. B 43. A 63. C* 83. A
4. B* 24. C 44. D 64. C 84. A
5. A 25. C 45. C 65. A 85. A
6. B 26. C 46. A* 66. B 86. B
7. C* 27. A 47. B 67. C 87. D
8. D 28. A 48. A 68. A 88. B
9. C* 29. A 49. B 69. D 89. B
10
E* 30. A 50. A 70. D* 90. D*
.
11
C* 31. D 51. C 71. D 91. D
.
12
D* 32. C 52. A 72. C 92. B
.
13
D* 33. A 53. B 73. C 93. D
.
14
D* 34. B 54. D 74. B 94. A
.
15
A* 35. E 55. B 75. B 95. C
.
16
B 36. B 56. A 76. B 96. C
.
17
D* 37. C 57. A 77. A 97. A
.
18
B 38. A 58. A 78. C 98. D*
.
19
A 39. A 59. A 79. D 99. D
.
20 100
A 40. D 60. A 80. A A
. .

*Note: Items with asterisk pertain to questions where the common answer of the examinees is different from the
correct answer indicating topics where examinees either find difficult, confusing, or are not familiar with. Review or
revisit those topics in your final preparation for the LECPA.

Exam insights:

Highest Score: 91%


Average Score: 53%

Reviewer’s Feedback:

There were major additions of topics in the syllabus of RFBT beginning October 2022 LECPA; hence, many examinees
were still unfamiliar with those newer topics.

Page 4 of 52
Personal copy of Barbin, Rafael R. (rafaelbarbin123@gmail.com)
Global Reciprocal Colleges,Inc-Caloocan

FINANCIAL ACCOUNTING AND REPORTING

1. D* 21. A 41. B 61. D*


2. C* 22. B 42. B* 62. D
3. D 23. C 43. C 63. A
4. D* 24. C 44. C 64. C
5. B 25. D* 45. C 65. C*
6. A 26. C 46. B 66. B*
7. C 27. D 47. B* 67. C
8. A 28. A 48. C 68. B*
9. C 29. C 49. B 69. B
10. D* 30. D 50. B* 70. C
11. C 31. B 51. C
12. A 32. A 52. B*
13. C 33. D 53. A*
14. C* 34. B 54. A
15. B 35. D* 55. C
16. B 36. C 56. B
17. A 37. B 57. B
18. B 38. A* 58. C
19. A 39. D* 59. B
20. B 40. C 60. C

*Note: Items with asterisk pertain to questions where the common answer of the examinees is different from the
correct answer indicating topics where examinees either find difficult, confusing, or are not familiar with. Review or
revisit those topics in your final preparation for the LECPA.

Exam insights:

Highest Score: 90%


Average Score: 41%

Reviewer’s Feedback:

Some of the items noted in asterisk above are difficult because these are:
• Multidisciplinary problems requiring application of concepts across different topics in FAR;
• Tricky; and
• Original (Not familiar).

However, these are answerable if the examinees have strong conceptual foundation.

Also, some errors are due to 'carelessness'. So, read and understand carefully the requirements and the given
information.

Page 5 of 52
Personal copy of Barbin, Rafael R. (rafaelbarbin123@gmail.com)
Global Reciprocal Colleges,Inc-Caloocan

ADVANCED FINANCIAL ACCOUNTING AND REPORTING

1. B 21. D 41. C 61. B


2. B 22. B 42. A 62. A*
3. C 23. B 43. A 63. D
4. C 24. A 44. Bonus^ 64. D
5. A 25. A 45. C 65. C
6. A 26. D 46. B 66. C*
7. C 27. D 47. B 67. A
8. A 28. C 48. A 68. C
9. A 29. C 49. A 69. B
10. C 30. A 50. D 70. D
11. B 31. A 51. D*
12. A 32. D 52. A
13. A 33. A 53. A
14. A 34. B 54. D
15. A 35. A 55. B
16. C 36. D 56. B
17. D 37. B 57. A*
18. D 38. A/C/D 58. D
19. C 39. D 59. C*
20. B 40. B 60. B*

*Note: Items with asterisk pertain to questions where the common answer of the examinees is different from the
correct answer indicating topics where examinees either find difficult, confusing, or are not familiar with. Review or
revisit those topics in your final preparation for the LECPA.

^Bonus due to inadequate information to answer the problem.

Exam insights:

Highest Score: 96%


Average Score: 52%

Reviewer’s Feedback:

Examinees tend to have difficulty answering original and/or tricky questions. Also, some topics are inherently
complicated such as accounting for special transactions and accounting for business combinations.

Page 6 of 52
Personal copy of Barbin, Rafael R. (rafaelbarbin123@gmail.com)
Global Reciprocal Colleges,Inc-Caloocan

Suggested Solution – MS

MC No. 1

MC No. 2
Unit contribution margin 2.30
Divide by: Unit selling price 5.50
Contribution margin ratio 41.82%
Multiplied by: Break-even point in pesos 150,500
Fixed costs 62,936

Net income 27,000


Fixed costs 62,936
Total contribution margin 89,936
Divide by: Unit contribution margin 2.30
Unit sales 39,103
Multiplied by: Unit selling price 5.50
Sales revenue 215,065

MC No. 3

MC No. 5
Ratio Total
Sales 100 50,000
%
Less: Variable costs 60% 30,000
Contribution margin 40% 20,000

Increase in net income [20,000-15,000] 5,000


Divide by: Contribution margin ratio 40%
Increase in peso sales 12,50
0

MC No. 6
Variable selling expense [18,000*5] 90,000
Fixed overhead [20,000*1] 20,000
Fixed selling and administrative expenses 40,000
Total unavoidable costs 150,000

Page 7 of 52
Personal copy of Barbin, Rafael R. (rafaelbarbin123@gmail.com)
Global Reciprocal Colleges,Inc-Caloocan

MC No. 8
Standard Delux
e
Selling price per unit 900 2,000
Less: Variable costs per unit
Direct materials per unit 100 500
Direct labor per unit 300 700
Variable overhead per unit 50 100
Contribution margin per unit 450 700
Divide by: Machine hours per unit 4 8
Contribution margin per machine 112.5 87.5
hour
Maximum total contribution margin [10,000*112.5] 1,125,00
0

MC No. 10
Formula price [1,920,000*1.10] 2,112,000
Additional [(2,200,000-2,112,000)*1/2] 44,000
Proceeds from the contract 2,156,000

MC No. 14
Current assets 1,200,000
Divide by: Current ratio 1.50
Current liabilities 800,000

Current liabilities 800,000


Multiplied by: Quick ratio 1.10
Quick assets 880,000

Current assets 1,200,000


Less: Quick assets 880,000
Inventory 320,000

Costs of goods sold [10,000,000*35%] 3,500,000


Divide by: Inventory 320,000
Inventory turnover 10.94

MC No. 16
Avoidable fixed manufacturing overhead 140,000
[(280,000-210,000)*2]
Avoidable fixed selling expenses [(88,000*0.10)*2] 17,600
Less: Start-up costs 4,000
Net avoidable fixed costs 153,600
Less: Lost contribution margin
Unit sales [26,000*2] 52,000
Multiplied by: Contribution margin per unit 4 208,000
[46-42]
Advantage (Disadvantage) of closing (54,400
)

Page 8 of 52
Personal copy of Barbin, Rafael R. (rafaelbarbin123@gmail.com)
Global Reciprocal Colleges,Inc-Caloocan

MC No. 17
Avoidable fixed manufacturing overhead 140,000
[(280,000-210,000)*2]
Avoidable fixed selling expenses [(88,000*0.10)*2] 17,600
Less: Start-up costs 4,000
Net avoidable fixed costs 153,600
Less: Lost contribution margin
Unit sales [26,000*2] 38,400
Multiplied by: Contribution margin per unit 4 153,600
[46-42]
Advantage (Disadvantage) of closing -

MC No. 20
Total Ratio
Sales 10,000,000 100%
Less: Variable
costs
Costs of goods sold 6,000,000
Commissions 2,000,000 8,000,000 80%
Contribution margin 2,000,000 20%
Less: Fixed costs 100,000
Income before taxes 1,900,000

Fixed costs 100,000


Divide by: Contribution margin ratio 20%
Break-even point in pesos 500,000

MC No. 21
Salary of full-time sales personnel [30,000*3] 90,000
Salary of sales manager 160,000
All other expenses (fixed) 100,000
Total fixed costs if the company employs its own sales force 350,000

Current commission of the existing sales agents [2,000,000/10,000,000] 20%

Current contribution margin ratio 20%


Deletion of the commission of existing sales agents 20%
Commission rate if the company employs its own sales personnel (5%)
Revised contribution margin ratio if the company employs its own sales force 35%

Fixed costs 350,000


Divide by: Contribution margin ratio 35%
Break-even point in pesos 1,000,000

MC No. 22
Current contribution margin ratio 20%
Additional commission of existing sales agents [25%-20%] (5%)
Revised contribution margin ratio 15%

Desired income before taxes 1,900,000


Fixed costs 100,000
Total contribution margin 2,000,000
Divide by: Contribution margin ratio 15%
Required sales in pesos 13,333,33
3

MC No. 23
Profit (if the company employs its own sales force) 0.35X-350,000
Profit (Existing sales agents at 25% commission) 0.15X-100,000

0.35X – 350,000 = 0.15X – 100,000


0.35X – 0.15X = 350,000 – 100,000
0.20X = 250,000
0.20
X = 1,250,000

Page 9 of 52
Personal copy of Barbin, Rafael R. (rafaelbarbin123@gmail.com)
Global Reciprocal Colleges,Inc-Caloocan

MC No. 26
Common stockholders’ equity, beginning [300,000+75,000] 375,000
Common stockholders’ equity, ending [400,000+185,000] 585,000
Total contribution margin 960,000
Divide by: 2
Average common stockholders’ equity 480,000

Net income to common [120,000-10,000] 110,000


Divide by: Average common stockholders’ equity 480,000
Break-even point in pesos 23%

MC No. 28
Per unit Total
Sales 18 360,00
0
Less: Variable costs 11 220,00
0
Contribution margin 7 140,00
0

Sales [(20,000*1.15)*18] 414,000


Less: Variable costs [(20,000*1.15)*12] 276,000
Contribution margin 138,000
Less: Fixed costs [105,000+19,200] 124,200
Net income 13,800
Less: Existing net income 35,000
Effect on net income (21,200
)

MC No. 32
Expected value of profit [(500,000*0.40)+(200,000*0.25)-(50,000*0.35)] 232,500

MC No. 33
Cost pools Budgeted costs Activity drivers Activity rate
[A] [B] [C=A/B]
Setup 80,000 100 800/setup
Ordering 40,000 500 80/order
Maintenance 100,000 2,500 40/machine hour
Power 20,000 5,000 4/kilowatt hour

Setup costs [2*800] 1,600


Ordering costs [5*80] 400
Maintenance costs [25*40] 1,000
Power costs [50*4] 200
Overhead costs of product X 3,20
0

MC No. 34
2022 2021
Sales 5,520,000 4,000,000
Less: Costs of sales 3,795,000 3,000,000
Gross profit 1,725,000 1,000,000

Sales at 2021 price [5,520,000/1.20] 4,600,000

Increase in sales due to change in selling price [5,520,000-4,600,000] 920,000


Increase in sales due to change in units sold [4,600,000-4,000,000] 600,000

Page 10 of 52
Personal copy of Barbin, Rafael R. (rafaelbarbin123@gmail.com)
Global Reciprocal Colleges,Inc-Caloocan

MC No. 35
2022 2021
Sales 5,520,000 4,000,000
Less: Costs of sales 3,795,000 3,000,000
Gross profit 1,725,000 1,000,000

Costs at 2021 price [3,795,000/0.96] 3,953,125


Percentage of increase in units sold [(3,953,125/3,000,000)-1] 31.77%
Sales at 2021 price [4,000,000*1.3177] 5,270,800

Increase in sales due to change in quantity [5,270,800-4,000,000] 1,270,800


Decrease in costs due to change in unit costs [3,953,125-3,795,000] 158,125

MC No. 36
Orchestra 15,000
Tickets and advertising 7,000
Auditorium rental 48,000
Floor show and strolling entertainment 10,000
Total fixed costs 80,000
Divide by: No. of attendees 250
Required contribution margin per person to break-even 320
Variable costs per person [70+30] 100
Required ticket price 420

MC No. 37
Budgeted total minutes [200,000*5] 1,000,000
Divide by: No. of minutes in an hour 60
Budgeted direct labor hours 16,667
Multiplied by: Budgeted variable overhead per direct labor 30
hour
Budgeted total variable overhead 500,000
Budgeted total fixed overhead [50,000*12] 600,000
Budgeted total overhead for the coming year 1,100,000

MC No. 38
Required minimum price 5.60 squeezed
Less: Variable costs per unit 4
Required contribution margin per unit 1.60
Multiplied by: No. of units of special order 10,000
Increase in contribution margin from special order 16,000
Less: Increase in fixed costs 10,000
Increase in profit 6,000
Less: Increase in required return on assets [40,000*15%] 6,000
Effect on residual income -

MC No. 39
Weighted average costs of capital [(5%*0.50)+(15%*0.50)] 10%

Net operating profit after taxes [300,000,000*0.60] 180,000,000


Less: Costs of capital sources [1,200,000,000*0.10] 120,000,000
Economic value-added 60,000,000
MC No. 40
Inventory, 12/31/2022 42,000
Costs of goods sold in 2022 300,000
Less: Inventory, 12/31/2021 30,000
Purchases in 2022 312,000

Accounts payable, 12/31/2021 20,000


Purchases in 2022 312,000
Less: Accounts payable, 12/31/2022 [312,000/12] 26,000
Budgeted payments in 2022 306,000

Page 11 of 52
Personal copy of Barbin, Rafael R. (rafaelbarbin123@gmail.com)
Global Reciprocal Colleges,Inc-Caloocan

MC No. 41
Product A
Sales price variance = (Actual selling price – Standard selling price)*Actual quantity sold
Sales price variance = (6.20–6.00)*8,000
Sales price variance = 1,600 favorable

Product B
Sales price variance = (Actual selling price – Standard selling price)*Actual quantity sold
Sales price variance = (7.70–8.00)*33,000
Sales price variance = 9,900 unfavorable

Product A 1,600 favorable


Product B 9,900 unfavorable
Total sales price variance 8,300 unfavorable

MC No. 44

MC No. 45
Interest expense 20,000
Multiplied by: Times interest earned ratio 4.5
Earnings before interest and taxes 90,000
Less: Interest expense 20,000
Earnings before taxes 70,000
Less: Income tax expense [40%] 28,000
Net income 42,000

MC No. 46
Total costs 56,000
Less: Variable costs [210,000*10%] 21,000
Fixed costs 35,000

Variable costs [168,000*10%] 16,800


Fixed costs 35,000
Total costs 51,800

MC No. 48
No. of units Average time per unit Total time
4 200 800 [4*200]
8 [4*2] 160 [200*0.80] 1,280 [8*160]
16 [8*2] 128 [160*0.80] 2,048 [16*128]

MC No. 51
Actual sales commissions 182,700
Less: Static budget for sales commissions [29,000*6] 174,000
Fixed costs 8,700U

MC No. 52
Selling price if processed into sweater 300
Selling price of the wool yarn 200
Increase in revenue 100
Less: Increase in costs
Buttons, thread, lining 20
Direct labor 58 78
Increase in profit if processed further 22
MC Nos. 53 &54
Selling price of the wool yarn (opportunity costs) 200
Less: Incremental costs of further processing
Buttons, thread, lining 20
Direct labor 58 78
Minimum price of the sweater 278

Page 12 of 52
Personal copy of Barbin, Rafael R. (rafaelbarbin123@gmail.com)
Global Reciprocal Colleges,Inc-Caloocan

MC No. 55
June July
Beginning inventory - 10,000
Production 100,000 100,000
Sales 90,000 105,000
Ending inventory 10,000 5,000

Fixed overhead costs 600,000


Divide by: Production 100,000
Fixed overhead per unit 6

July
Absorption net income 20,000 squeezed
Fixed overhead in the beginning inventory [10,000*6] 60,000
Less: Fixed overhead in the ending inventory [5,000*6] 30,000
Variable net income 50,000

MC No. 56
Labor rate variance = (Actual rate – Standard rate)*Actual hours
Labor rate variance = (16–15)*2,500
Labor rate variance = 2,500 unfavorable

Labor efficiency variance = (Actual hours-Standard hours)*Standard rate


Labor efficiency variance = (2,500-2,250)*15
Labor efficiency variance = 3,750 unfavorable

Standard hours [150*15] 2,250

MC No. 57
Materials price variance = (Actual price – Standard price)*Actual quantity purchased
Materials price variance = (12–15)*3,100
Materials price variance = (9,300) favorable

Materials quantity variance = (Actual quantity – Standard quantity)*Standard price


Materials quantity variance = (3,100–3,000)*15
Materials quantity variance = 1,500 unfavorable

Standard quantity [150*20] 3,000

MC No. 59
No. of shares 226,087 squeezed
*Net issue price per share [25*0.92] 23
Proceeds from sale 5,200,000
Less: Stock issue costs 200,000
Net proceeds 5,000,000

MC No. 60
After tax costs of debt [9%*0.65] 5.85%

Acquisition costs 62,000


Less: Present value of depreciation tax shield [(62,000/3)*0.35*2.6804] 19,388
Net costs of buying 42,612
Less: Costs of leasing
PV of lease payments [(22,500*0.65)*2.6804] 39,201
Net advantage to leasing 3,411

MC No. 62
Required return = Risk-free rate + [Beta * Market risk premium]
0.1375 = 0.05 + 0.07X
0.1375 – 0.05 = 0.07X
0.0875 = 0.07X
0.07
X = 1.25

Page 13 of 52
Personal copy of Barbin, Rafael R. (rafaelbarbin123@gmail.com)
Global Reciprocal Colleges,Inc-Caloocan

MC No. 63
Amount Capital
structure
Debt 2,000,000 66.67% or 2/3
Equity 1,000,000 33.33% or 1/3
Total 3,000,000 100%

After tax costs of debt [8%*0.60] 4.8%

Weighted average costs of capital = [Costs of debt after tax*Weight of debt] + [Costs of equity*Weight of equity]
0.12 = [0.048*2/3] + 0.33333333333X
0.12 = 0.032 + 0.33333333333X
0.12 – 0.032 = 0.33333333333X
0.088 = 0.33333333333X
0.33333333333
X = 26.4%

MC No. 64
State of Economy Rate of Probability of Expected value of return
return occurrence
Boom 25% 0.45 11.25%
Normal 15% 0.50 7.5%
Recession 5% 0.05 0.25%
Expected value of 19%
return

State of Rate of Expected Deviation Squared Probability of Variance [F


Economy return [A] value of [C = Deviation [D = occurrence = D*E]
return [B] A-B] C*C] [E]
Boom 25 19 6 36 0.45 16.2
Normal 15 19 -4 16 0.50 8
Recession 5 19 -14 196 0.05 9.8
Total 34
variance

Standard deviation is computed as the square root of the total variance.


Standard deviation [square root of 34] 5.83095

Standard deviation 5.83095


Divide by: Expected return 19
Coefficient of variation 0.3069

MC No. 66
Total financing [75M-15M] 60M

Source of Financing Capital Amount of


structure financing
Debt 25% 15,000,000
Preferred Equity 5% 3,000,000
Common Equity 70% 42,000,000
Total financing 100% 60,000,000

% of the 75M to come from the issue of new common stock [42M/75M] 56%

MC No. 67
Increase in receivables [(720,000/360)*75] 150,000
Multiplied by: Variable costs ratio 80%
Investment in receivable 12,000
Multiplied by: Opportunity costs rate 20%
Carrying costs of receivable 24,000

Increase in contribution margin [720,000*20%] 144,000


Less: Increase in carrying costs of receivable 24,000
Benefit (Loss) from relaxation of credit terms 120,000

Page 14 of 52
Personal copy of Barbin, Rafael R. (rafaelbarbin123@gmail.com)
Global Reciprocal Colleges,Inc-Caloocan

MC No. 69
Acquisition costs of the new machine 800,000
Trade-in value of the old machine (60,000)
After tax avoidable costs of repairs [200,000*0.65] (130,000)
Net investment 610,000

MC No. 70
Expected revenue 40,000
Expected cash expenses 29,000
Expected cash income before taxes 11,000
Less: Income tax expense [40%] 4,400
Expected cash income after taxes 6,600
After tax proceeds from sale [9,000*0.60] 5,400
Proceeds from disposal of working capital [7,000+5,000] 12,000
Expected net cash flow in the tenth year 24,000

Page 15 of 52
Personal copy of Barbin, Rafael R. (rafaelbarbin123@gmail.com)
Global Reciprocal Colleges,Inc-Caloocan

Suggested Solution – AUD

1. C

The objective of an audit of financial statements is to obtain reasonable assurance whether the financial statements
are free from material misstatements, whether due to fraud or error, to enable the auditor to express an opinion as to
the fairness of the financial statements, in all material respects, in accordance to the applicable financial reporting
framework and to communicate such opinion to the users of the financial statements. Choice (A) is incorrect
because the types and amount of evidence may vary from audit to audit. Choice (B) is incorrect because the criteria
may vary depending on the information being evaluated. Choice (D) is incorrect because the criteria where the
auditor compares the audited information is not limited to SEC standards.

2. D

Answer (D) is correct because the professional standards require that when the predecessor auditor’s opinion was
other than unmodified, the successor should describe the nature of any reasons for the qualification. Answer (A) is
incorrect because reference should be made to the other statements. Answer (B) is incorrect because division of
responsibility (if allowed by the jurisdiction in which the auditor practices) in an audit report applies only when a
portion of 1 year’s examination has been performed by other auditors. Answer (C) is incorrect because written
approval from the predecessor need not be obtained in order to include the prior year’s financial statements.

3. D

Answer (D) is correct. Auditors ordinarily will not discuss with management the details of procedures that are
necessary to perform the audit. Answers (A), (B), and (C) are incorrect. Engagement letters may include information
on fees. Engagement letters may include information on observed opportunities for economy. Engagement letters will
include a statement on the risk of not detecting material errors and fraud.

4. D

Answer (D) is correct because an auditor applies limited procedures and reports deficiencies or omission of such
information; note that while deficiencies are reported in the audit report, the opinion is not modified since the
information is considered unaudited. Answer (A) is incorrect because it is seldom the case that a specialist will be
utilized because the auditor is not providing an opinion on the supplemental information. Answer (B) is incorrect
because an auditor should apply certain limited procedures beyond determining that the information has not been
omitted. Answer (C) is incorrect because tests of transactions need not be performed.

5. D

Answer (D) is correct. Fraud is a broad legal concept and auditors do not make legal determinations of whether fraud
has occurred. Answer (A) is incorrect. Considering journal entries is an ordinarily required audit procedure to address
the risk of management override of controls. Answer (B) is incorrect. The review of accounting estimates for biases is
an ordinarily required audit procedure to address the risk of management override of controls. Answer (C) is incorrect.
The evaluation of the business rationale for significant unusual transactions is an ordinarily required audit procedure
to address the risk of management override of controls.

6. D

Answer (D) is correct because a compilation includes a consideration of whether financial statements are free from
obvious material misstatements. Answer (A) is incorrect. This answer is incorrect because such inquiries are not
required. Answer (B) is incorrect. This answer is incorrect because a consideration of going concern status is not
required in a compilation. Answer (C) is incorrect. This answer is incorrect because scanning the records after the
balance sheet date is not required.

7. B

Answer (B) is correct. This answer is correct because such deficiencies suggest the possibility of misstatement, and
accordingly suggest the need for an increase in the assessment of control risk, with a corresponding increase in the
scope of substantive procedures. Answer (A) is incorrect. This answer is incorrect because the significant deficiencies,
by themselves, need not necessarily result in a disclaimer of opinion, although such an opinion is possible if the
auditor determines that a very significant scope limitation is involved. Answer (C) is incorrect. This answer is incorrect
because significant deficiencies, by themselves, need not necessarily result in a qualified opinion, although a qualified
opinion is possible if the auditor determines that a scope limitation is involved. Answer (D) is incorrect. This answer is
incorrect because significant deficiencies may, but generally do not, require withdrawal from the engagement.

Page 16 of 52
Personal copy of Barbin, Rafael R. (rafaelbarbin123@gmail.com)
Global Reciprocal Colleges,Inc-Caloocan

8. A

Answer (A) is correct. Discovery sample sizes and related discovery sampling tables are constructed to measure the
probability of at least one error occurring in a sample if the error rate in the population exceeds the tolerable rate.
Answer (B) is incorrect. Variables sampling need not include at least one exception (mean per unit sampling, for
example, needs no errors). Answer (C) is incorrect. Random sampling only deals with the technique used to select
items to be included in the sample. Answer (D) is incorrect. Mean-per-unit sampling results are not directly related to
finding at least one exception.

9. D

Answer (D) is correct. This answer is correct because audit workpapers generally include the manner in which
exceptions and unusual matters were resolved. The workpapers should contain support for the auditor’s conclusions
concerning significant aspects of the examination. Answer (A) is incorrect. This answer is incorrect because the
personal financial status of the client’s management team is not relevant to the fairness of the client company’s
financial statements. Answer (B) is incorrect. This answer is incorrect because audit workpapers may serve as a
useful reference to the client but may not be regarded as a part of, or a substitute for, the client’s accounting records.
Answer (C) is incorrect. This answer is incorrect because reference to pertinent authoritative pronouncements in the
audit workpapers is sufficient to support compliance with GAAP, excerpts from the pronouncements need not be
provided.

10. B

Answer (B) is correct. Inverse is correct because as control risk increases (decreases) detection risk must decrease
(increase).

11. A

Answer (A) is correct. This answer is correct because the professional standards indicate that the CPA may receive
assistance in performing both tests of controls and substantive procedures.

12. A

Answer (A) is correct. Lapping will result in a delay in the recording of specific remittance credits on the financial
records, but the checks will be deposited in the bank as they are received. Answer (B) is incorrect. The daily cash
summaries will include the same sums as the cash receipts journal entries. Answer (C) is incorrect. The bank deposit
slips will be identical to any details included in the monthly bank statements. Answer (D) is incorrect. While the
write-off of a receivable may help the individual involved in the lapping to avoid repayment, no lag is to be expected
between authorization of the write-off and the date it is actually recorded.

13. B

Answer (B) is correct. The automatic updating of payroll records alters the audit trail which, in the past, included
steps pertaining to manual updating. Answer (A) is incorrect. Although an auditor may choose to use a generalized
computer audit program, it is not required. Answer (C) is incorrect. No information is presented that would
necessarily indicate a change in the likelihood of fraud. Answer (D) is incorrect. Given automatic updating, a large
portion of the transactions are not processed in batches.

14. A

Answer (A) is correct. This answer is correct because that information should be marked "unaudited." Answer (B) is
incorrect. This answer is incorrect because segment information is required to be disclosed. Answer (C) is incorrect.
This answer is incorrect because the notes are considered audited. Answer (D) is incorrect. This answer is incorrect
because the auditor should audit investment security classifications to obtain the valuation objective.

15. A

Answer (A) is correct. This answer is correct because the measure of the validity of evidence for audit purposes is
based upon the judgment of the auditor. This audit judgment is used to estimate levels of materiality, and relative
risk. Answer (B) is incorrect. This answer is incorrect because estimating levels of materiality is only one aspect of
audit judgment used to determine necessary auditing procedures. Answer (C) is incorrect. This answer is incorrect
because evaluating relative risk is only one aspect of audit judgment used to determine necessary auditing
procedures. Answer (D) is incorrect. This answer is incorrect because while auditors obtain reasonable assurance of
detecting misstatements, this is not what ultimately determines what specific audit procedures that are necessary.

Page 17 of 52
Personal copy of Barbin, Rafael R. (rafaelbarbin123@gmail.com)
Global Reciprocal Colleges,Inc-Caloocan

16. C

Answer (C) is correct. A distributed data processing system links minicomputers in remote locations with a centralized
computer. The controls of greatest concern to the auditor are access controls because each minicomputer will be able
to access the central computer and it will be more difficult to control access to minicomputers in remote locations.
Answer (A) is incorrect. Hardware controls pertain to controls over physical access to computers, printers, modems,
disk drives, etc. They would not be of as much concern to the auditor in a distributed data processing system as
access controls. Answer (B) is incorrect. Systems documentation controls ensure that adequate documentation is
maintained for each application system. Such documentation includes memos, flowcharts, and program listings.
Systems documentation controls would not be of as much concern to the auditor in a distributed data processing
system as access controls. Answer (D) is incorrect. Disaster recovery controls pertain to the entity's ability to become
operational in the event of a major disaster. They would not be of as much concern to the auditor in a distributed data
processing system as access controls.

17. B

Answer (B) is correct. Philippine Standards on Related Services state that additional necessary audit effort and the
reason for the change—as well as the additional cost to complete the audit—be considered.

18. D

Answer (D) is correct. This answer is correct because the professional standards do not allow such a statement.
Answer (A) is incorrect. This answer is incorrect because adverse opinions use a phrase such as "because of the
significance of the matter…". Answer (B) is incorrect. This answer is incorrect because a qualified opinion ordinarily
indicates "except for..." Answer (C) is incorrect. This answer is incorrect because an unmodified opinion with a
separate emphasis-of-matter paragraph includes no modification of the opinion paragraph.

19. D

Answer (D) is correct. This answer is correct because very early in the engagement the auditor should determine how
much assistance is needed of the client’s personnel, and should coordinate that assistance. Answer (A) is incorrect.
This answer is incorrect because auditors do not ordinarily emphasize the appropriateness of the entity’s dealing with
adverse economic conditions— although this may occur when a question concerning going concern status occurs.
Answer (B) is incorrect. This answer is incorrect because the determination of fraud risk factors will occur
subsequently. Answer (C) is incorrect. This answer is incorrect because control weaknesses will be identified
subsequently.

20. C

Answer (C) is correct. A bill and hold transaction results in the recording of a sale prior to delivery of the
goods—accordingly, sales may be inappropriately recorded. Answer (A) is incorrect. Accrued liabilities are not
ordinarily affected by bill and hold transactions. Answer (B) is incorrect. An absolute purchase commitment represents
a condition that increases the likelihood that recording of a sale for such a transaction is appropriate. Answer (D) is
incorrect. The assuming of risk and reward relating to the product represents a condition that increases the likelihood
that recording of a sale for such a transaction is appropriate.

21. A
Answer (A) is correct. This answer is correct because either a qualified opinion or a disclaimer of opinion due to a
scope limitation is appropriate when the auditor is unable to obtain the audited financial statements of a significant
subsidiary. Answer (B) is incorrect. This answer is incorrect because an unmodified opinion may be issued on income
tax basis statements. Answer (C) is incorrect. This answer is incorrect because an unmodified opinion may be
appropriate when there are significant deficiencies in internal control if the auditor is able to perform adequate
substantive procedures. Answer (D) is incorrect. This answer is incorrect because other procedures may be
appropriate when analytical procedures indicate that the current year balances are not comparable to prior years.

22. C

Answer (C) is correct. This answer is correct because a lack of approval may lead to a question as to the collectibility
of receivables (valuation); the credit approval policies do not directly relate to the rights and obligations assertion.

23. C

Answer (C) is correct. This answer is correct because professional skepticism is not an element of quality control. This
is an individual auditor’s mindset. Answers (A), (B), and (C) are elements of quality control. See PSQC 1.

Page 18 of 52
Personal copy of Barbin, Rafael R. (rafaelbarbin123@gmail.com)
Global Reciprocal Colleges,Inc-Caloocan

24. D

Answer (D) is correct. This answer is correct because the auditor would appropriately conclude that entries in the
accounts receivable subsidiary ledger are properly supported by sales invoices. Answer (A) is incorrect. This answer is
incorrect because the auditor would trace sales invoices to supporting bills of lading in order to conclude that sales
invoices represent bona fide sales. Answer (B) is incorrect. This answer is incorrect because the auditor would trace
bills of lading to entries in the subsidiary ledger in order to conclude that all sales have been properly recorded.
Answer (C) is incorrect. This answer is incorrect because the auditor will trace sales invoices to the subsidiary ledger
in order to conclude that sales invoices have been properly posted to customer accounts.

25. B

The requirement is to identify an advantage of using systems flowcharts to document information about internal
control instead of using internal control questionnaires. Answer (B) is correct because flowcharts provide a visual
depiction of clients' activities which make it possible for auditors to quickly understand the design of the system.
Answer (A) is incorrect because while the flow of operations is visually depicted, internal control weaknesses are not
as obvious. Answer (C) is incorrect because while a flowchart describes a system, the flowchart alone does not
indicate whether that system is operating effectively. Answer (D) is incorrect because auditors still need to determine
whether the system has been placed in operation and therefore the need to observe employees performing routine
tasks remains.

26. C

Answer (C) is correct. Information such as this is ordinarily included in a management representation letter. See PSA
580 for guidance on representation letters. Answer (A) is incorrect. Such a disclosure is not ordinarily included in a
report on internal control. Answer (B) is incorrect. Such a disclosure is not ordinarily included in a special report.
Answer (D) is incorrect. Such a disclosure is not ordinarily included in a letter for an underwriter.

27. B

Answer (B) is correct. This answer is correct because the professional standards suggest the use of the positive form
for large balances and the negative form for small balances. Answer (A) is incorrect. This answer is incorrect because
it is unlikely the auditor would confirm all balances. Answer (C) is incorrect. This answer is incorrect because there is
no requirement that the positive form be used for trade receivables or that the negative form be used for other
receivables. Answer (D) is incorrect. This answer is incorrect because positive confirmations are suggested when
internal control is unsatisfactory and the negative form is suggested when internal control is satisfactory.

28. D

Answer (D) is correct. This answer is correct because it is based only on preliminary information and may well be
satisfactorily resolved. Answers (A), (B), and (C) are incorrect. This answer is incorrect because the professional
standards require that disagreements that should be communicated include those relating to estimates, the scope of
the audit, application of accounting principles, the wording of the audit report, and other matters.

29. D

Answer (D) is correct. This answer is correct because permanent files include information affecting more than 1 year,
such as analyses of capital stock and other owners’ equity accounts. While these accounts include few transactions
annually, they have substantial balances which carry over to each following year. Answer (A) is incorrect. This answer
is incorrect because schedules that support the current year’s adjusting entries will normally be retained in the
current year files because of their direct relationship only to the current year audit. Answer (B) is incorrect. This
answer is incorrect because the high turnover of receivables result in a situation in which receivable confirmation
replies provide evidence typically related to only the year under audit. Answer (C) is incorrect. This answer is
incorrect because most documentation on the planning and supervision of each year’s audit will be retained in that
year’s current files.

30. C

Answer (C) is correct. This answer is correct because when the financial statements are affected by a departure from
generally accepted accounting principles, the auditor will express a qualified opinion. When expressing a qualified
report, the auditor should disclose all the substantive reasons in a “Basis for Qualified Opinion” section. Answer (A) is
incorrect. This answer is incorrect because a qualified opinion is not required if the auditor conducts the examination
per GAAS and applies all procedures necessary in the circumstances. Here the auditor is satisfied with alternative
procedures. Answer (B) is incorrect. This answer is incorrect because this would generally not lead to a qualified
opinion due to an immaterial change in accounting principle. Answer (D) is incorrect. This answer is incorrect because
a reference to another auditor's report does not constitute a qualification of the auditor's report.

Page 19 of 52
Personal copy of Barbin, Rafael R. (rafaelbarbin123@gmail.com)
Global Reciprocal Colleges,Inc-Caloocan

31. C

The requirement is to identify the control that will be most effective in offsetting the tendency of sales personnel to
maximize sales volume at the expense of high bad debt write-offs. Answer (C) is correct because segregation of the
authorization of credit from the sales function will allow an independent review of the creditworthiness of customers.
Answer (A) is incorrect because while denying access to cash by employees responsible for sales and bad debt
write-offs may deter embezzlements, the problem of high bad debt write-offs is likely to remain. Answer (B) is
incorrect because while so segregating the matching of shipping documents and sales invoices may help assure that
items are shipped properly and subsequently recorded, it will not significantly affect bad debts. Answer (D) is
incorrect because while independent reconciliation of control and subsidiary accounts receivable records may defer
embezzlements, bad debt write-offs will not be affected.

32. D

Answer (D) is correct. A high assessed level of control risk may result in changing the timing of substantive tests to
year-end rather than at an interim date. If the assessed level of control risk is low, the auditor could perform interim
substantive tests and rely upon internal control to provide valid year-end records. However, because the assessed
level of control risk is high, the controls cannot be relied upon. Also, the nature of substantive tests may change from
less effective to more effective procedures (e.g., use of independent parties outside the entity rather than internal)
and an increase in the extent of procedures (e.g., larger sample sizes). Answer (A) is incorrect. An auditor will
generally seek a year-end count of inventory. Answer (B) is incorrect. Gross profit tests will not in general have the
required precision when control risk is high. Answer (C) is incorrect. Tests of controls are likely to substantiate an
auditor's view that control risk is high, and it is therefore unlikely that their performance will be cost- effective.

33. C

Answer (C) is correct. This answer is correct because the professional standards state that in such circumstances a
standard unmodified audit report should be issued. Answer (A) is incorrect. This answer is incorrect because the
change is not referred to in the audit report. Answer (B) is incorrect. This answer is incorrect because the change is
not referred to in the audit report. Answer (D) is incorrect. This answer is incorrect because the change is not referred
to in the audit report.

34. D

Answer (D) is correct. The audit report date is the last day of the auditor’s responsibility for the review of significant
events that occurred after the date of the financial statements; hence, the date of the audit report is also referred to
as the last day of fieldwork of the auditor. Answer (A) is incorrect. The audit report date is dated later than the last
day of the fiscal period of the client. Answer (B) is incorrect. The audit report date does not necessarily coincide with
the the date on which the financial statements were filed with the SEC. Answer (C) is incorrect. Users may institute a
lawsuit against the client or auditor at a later date that the date of the audit report.

35. A

Answer (A) is correct. This circumstance is least likely to cause uncertainty about the ability if an entity to continue as
a going concern as the subject matter of the lawsuit concerns minor issue only. Answers (B), (C), and (D) are
examples of circumstances that are likely to cause uncertainty about the ability of an entity to continue as a going
concern.

Page 20 of 52
Personal copy of Barbin, Rafael R. (rafaelbarbin123@gmail.com)
Global Reciprocal Colleges,Inc-Caloocan

Problem no. 1

Question No. 39 - C
Cash and cash equivalents (No. 36) 4,780,000
FA at FVTPL 600,000
Trade and other receivables (No. 37) 11,420,000
Inventories (No. 38) 16,230,000
33,030,00
Total current assets 0

Question No. 43 - B
Cuurent assets (No. 39) 33,030,000
Property, plant and equipment (No. 40) 17,000,000
Investment property (No. 41) 3,000,000
Intangible assets (No. 42) 550,000
Other non-current assets 900,000
54,480,00
Total assets 0
PROBLEM NO. 2 - Aurelius Mining Corp.

Question No. 46 - C
Acquisition cost 16,640,000
Less residual value 1,280,000
Depletable cost 15,360,000
Divide by total estimated reserves 12,800,000
Depletion rate 1.20
Tons mined in 2022 1,280,000
Depletion for 2022 1,536,000

Question No. 47 - C
Depreciation - Building [(P1,280,000/12,800,000 tons)x1,280,000 tons x .8] 102,400
Depreciation - Machinery [(P2,560,000-P512,000/4] 512,000
Total 614,400

Page 21 of 52
Personal copy of Barbin, Rafael R.
(rafaelbarbin123@gmail.com)
Global Reciprocal Colleges,Inc-Caloocan

Question No. 48 - B
Depletion (See no. 46) 1,536,000
Direct labor 1,024,000
Depreciation (See no. 47) 614,400
Miscellaneous mining overhead 204,800
Total available for sale 3,379,200
Divide by tons mined 1,280,000
Cost per ton 2.64
Unsold tons (1,280,000 - 1,024,000) 256,000
Inventory, 12/31/22 675,840

Question No. 49 - B
Cost of sales (1,024,000 tons x P2.64) 2,703,360

Question No. 50 - B
Sales (1,024,000 x P4.4) 4,505,600
Less cost of sales (see no. 49) 2,703,360
Gross profit 1,802,240
Operating expenses (921,600)
Depreciation-Building [(P1,280,000/12,800,000 tons) x 1,280,000 tons x .2] (25,600)
Net income 855,040
Realized depletion (1,024,000 tons x P1.2) 1,228,800
Profit for 2022 (Maximum amount that may be declared as dividends) 2,083,840

PROBLEM NO. 3 - Seneca Corp.

Question No. 51 - C
August collections credited by bank (P10,000 +
P14,000) 24,000
Deposit in transit,
August 31 9,400
Unrecorded deposit in July recorded in August 1,000
Note collected by bank in July recorded in August 8,000
Unadjusted book receipts for August 42,400

Question No. 52 - B
Checks drawn in August (checks no. 455 to 460) 8,360
BSC for July recorded in August 100
Unadjusted book disbursements for
August 8,460

Note: Nos. 51 and 52 can alternatively be solved by working back from the adjusted bank balances.

Page 22 of 52
Personal copy of Barbin, Rafael R. (rafaelbarbin123@gmail.com)
Global Reciprocal Colleges,Inc-Caloocan

Seneca Corp.
Proof of Cash
For the month of August, 2022
August
July 31 Receipts Disb Aug. 31

Unadjusted bank balances 52,000 25,140 9,740 67,400


Deposits in transit:
July 900 (900)
August 9,400 9,400
Outstanding checks
July (4,000) (4,000)
August (check nos. 436, 457 and 460) 3,600 (3,600)
Erroneous bank debit, corrected (100) (100)
NSF check redeposited (140) (140)

Adjusted bank balances 48,900 33,400 9,100 73,200

Unadjusted book balances 40,000 42,400 8,460 73,940


Note collected by bank, July 8,000 (8,000)
Unrecorded deposit in July 1,000 (1,000)
Bank service charges
July (100) (100)
August 440 (440)
NSF check not redeposited 300 (300)

Adjusted book balances 48,900 33,400 9,100 73,200


48 - A 49 - B 50 - B

PROBLEM NO. 4 - Epectitus Corp.

Question No. 56 - B
Total proceeds 206,500
Less accrued interest (P200,000 x 9% x 5/12) 7,500
Net proceeds 199,000
Less cost of treasury notes:
Total amount paid 198,500
Less accrued interest (P200,000 x 9% x 3/12) 4,500 194,000
Gain on sale of treasury notes 5,000

Question No. 57 - D
Sales proceeds 70,000
Carrying amount (previous fair value) 90,000
Gain (loss) on sale of Vault shares (20,000)

Question No. 58 - D
Interest income on Ring Co. bonds (P300,000 x 12%) 36,000
Interest income on Phil. treasury notes (P200,000 x 9% x 8/12) 12,000
Total 48,000

Question No. 59 - C
Floor Textile shares (6,000 shares x P44) 264,000
Ring Co. Bonds (P300,000/P1,000 x P950) 285,000
Fair value, 12/31/22 549,000

Page 23 of 52
Personal copy of Barbin, Rafael R.
(rafaelbarbin123@gmail.com)
Global Reciprocal
Colleges,Inc-Caloocan

Question No. 60 - B
Fair value, 12/31/22 (See no. 59) 549,000
Carrying amount before FV adjustment, 12/31/22
Carrying amount, 12/31/21 640,600
Purchase of Phil. treasury notes, 4/1 194,000
Carrying amount of Vault, Inc. shares sold, 7/1 (90,000)
Sale of Phil. treasury notes, 12/1 (194,000) 550,600
Fair vaue adjustment gain (loss) (1,600)

PROBLEM NO. 5 - Cato Corp.

Question No. 61 - B
Cost of equipment (Cash price) 2,370,000

Question No. 62 - A
Question No. 63 - A
Int.
Date Payment (10%) Principal C.A.
1/1/21 2,070,000
207,0
12/31/21 388,020 00 181,020 1,888,980
188,8
12/31/22 388,020 98 199,122 1,689,858

Question No. 64 - D
Question No. 65 - A

PROBLEM NO. 6 - Zeno Corp.

Share Retained Treasury


Share capital premium earnings shares
Balances, 12/31/20 3,000,000 300,000 450,000 -
Issuance of CS, 6/15/21 5,000,000 1,000,000
Share dividend, 9/30/21 (See Note 1) 400,000 40,000 (440,000)
Profit - 2021 475,000
Balances, 12/31/21 8,400,000 1,340,000 485,000 -
Acquisition of TS, 3/1/22 285,000
5/31/22 (See Note 2) 37,500 (142,500)
9/15/22 (See Note 3) 5,000,000 1,250,000
10/31/22 (See Note 4) 8,000,000 2,000,000
12/10/22 (See Note 5) (425,000)
Retired 1,000 TS, 12/20/22 (100,000) 5,000 (95,000)
Profit - 2022 500,000
Balances, 12/31/22 21,300,000 4,632,500 560,000 47,500
66 - C 67 - D 68 - B

Notes:
1 Share dividend - Debit to RE [(30,000 + 50,000) x 5% x P110]
2 Share premium from TS [1,500 x (P120 - P95)]
3 Proceeds from exercise of rights (25,000 x 2 x P125)
4 Proceeds from exercise of rights (40,000 x 2 x P125)
Cash dividends [(84,000 + 50,000 + 80,000 - 1,500) x
5 P2]

Question No. 69 - C
Share capital (No. 66) 21,300,000
Share premium (No. 67) 4,632,500
Retained earnings (No. 68) 560,000
Treasury shares (47,500)
Total equity 26,445,000

Question No. 70 - D
Page 24 of 52
Page 25 of 52
Personal copy of Barbin, Rafael R. (rafaelbarbin123@gmail.com)
Global Reciprocal Colleges,Inc-Caloocan

Suggested Solution – TAX

1. C. Express provision of CREATE Law.


2. B. 5% is based on gross income.
3. B. Quarterly only.
4. A. 10% for building and 20% for machinery.
5. D. 1 year from date of death.
6. D.
7. D.
8. A. Basis is Section 232 of the Tax Code, as amended.
9. D. Exchange is tax “free”.
10. C. Basis is Revenue Memorandum Circular No. 6-2022
11. A. Service of eLOA within 30-day period has been removed.
12. C.
13. A. Express provision of TRAIN law.
14. C.
15. A.
16. A. SC Ruling on CIR vs QL Devt Corp GR 258947 dated March 29, 2022.
17. A. Express provision of CREATE Law
18. C.
19. D.
20. C. Constitution provides that we will adhere to progressive tax system as we want our taxation to be based on
ability to pay principle.
21. D. Specific provision of LGC Section 216 in relation to Sec 215.
22. D. 10% only for solo parents.
23. A.
24. C. BMBE is subject to percentage tax. Exemption is on income tax only
25. C. CIR vs. QL Dev’t Corp.
26. D. A is correct. VAT has prescribed for 1st quarter (deadline April 25, 2019 - prescription April 15, 2022) and 2nd
quarter (deadline July 25, 2019 – prescription July 25, 2022). B is correct. BIR can assess for 4th quarter
1601-EQ until January 31, 2022 (1st quarter deadline April 30, 2019 – prescription April 30, 2022; 2nd quarter
deadline July 31, 2019 – prescription July 31, 2022; 3rd quarter deadline October 31, 2019 – prescription October
31, 2022; 4th quarter deadline January 31, 2020 -prescription January 31, 2023.) C is correct. Prescription do not
run if no return is filed. D is wrong. February 27, 2022 is beyond the 30 day period to file a valid protest
27. A. Express provision of TRAIN law
28. C.
29. A/C. Technically accurate answer is A. Answer C may be considered as correct too given that if net estate exceeds
5M it follows that gross estate exceeds 5M.
30. D. 1 year from date of death.
31. D. BIR can issue assessment based on best evidence obtainable.
32. D. The 30-day period to serve the e-LOA was already removed.
33. C.
34. A. DME’s are not entitled to SCIT of 5% - only enhanced deductions.
35. D. S1 - 0% applies if goods are to be used directly and exclusively used in registered activity. S2 – 0% applies if
goods are to be used directly and exclusively used in registered activity. S3 – 12% if local purchases.
36. C. Laboratory tests are VAT-exempt.
37. C. S1 – VAT exempt regardless of amount. S2 – subject to 12% beginning January 1, 2021. S3 – VAT exempt
subject to ceiling. S4 – subject to 12% beginning January 1, 2021.
38. D.
39. D. excluded from excise tax are surgeries which aims to improve function and give a normal appearance to a part
of a person’s body that has been damaged, ameliorate a deformity arising from, or directly related to, a
congenital or developmental defect or abnormality, a personal injury resulting from accident or trauma, or
disfiguring disease, tumor, virus or infection.
40. C.
41. D. A – sale to government – 12%. B – Payment to non-resident – 12% (VAT withholding). C – Sale to DME –
12%. D – Rental below Php 15,000 is VAT-exempt regardless of total amount.
42. C.
43. C.
44. D.
45. D.
46. C.

Transaction Amount Rate DST


Subscribe capital stock 250,000 2.00/200 2,500
Loan 1,000,000 1.50/200 x6/12mos 3,750
Lease 5,000,000 Php 6 - 2,000 10,002
Php 2 in excess of 1,000
Total 16,252

Page 26 of 52
Personal copy of Barbin, Rafael R. (rafaelbarbin123@gmail.com)
Global Reciprocal Colleges,Inc-Caloocan

47. C. Idol Rappy is not qualified to use 8% preferential tax.

Itemized Deduction OSD


Gross receipts 30,000,000 30,000,000
Less: Deduction 5,000,000 12,000,000
Net income 25,000,000 18,000,000

Income tax due 8,360,000 5,910,000

48. C.

Foreign tax credit 1,000,000


EWT (VAT-registered - 10%) 25M 2,500,000
Quarterly tax payment 1,000,000
Total tax credit 4,500,000

49. C.

Income Payments Amount Rate EWT Due


Purchases 1,000,000 1% 10,000
Rent 120,000 5% 6,000
Utility 60,000 2% 1,200
Professional fee 120,000 10% 12,000
Total 29,200

50. A. Salary of each employee is less than Php 250,000 hence exempt.
51. A. Exempt due to registration as BMBE. If not BMBE, answer is P30,000.
52. A. Exempt due to registration as BMBE. If not BMBE, answer is P250,000.
53. A. Exempt due to registration as BMBE. If not BMBE, answer is 140,000.
54. B. 1% of Php 2,000,000.
55. D. The basis is the beginning inventory at transition hence the ending inventory is the amount to be used in
computing 2% transitional input tax (Php 200,000 x 2%).
56. D. Higher of 2% and actual input tax.
57. B.

1st Quarter
Gross sales 500,000
Less: 250,000
Taxable income 250,000
Rate 8%
Tax due 20,000

58. C. Covers tax paid on three quarters only. No 4th quarter in income tax.

Gross sales 1,800,000


Less: 250,000
Taxable income 1,550,000
Rate 8%
Tax paid for 3 quarters 124,000

59. C.

1st Quarter 2nd 3rd 4th Quarter Itemized OSD


Quarter Quarter
Gross sales 500,000 600,000 700,000 800,000 2,600,00 2,600,00
0 0
Cost of sales 200,000 400,000 400,000 600,000 1,600,00
(purchases) 0
Gross income 300,000 200,000 300,000 200,000 1,000,00
0
Expenses 100,000 100,000 100,000 100,000 400,000 1,040,00
0
Net income 200,000 100,000 200,000 100,000 600,000 1,560,00
0
Income tax due 80,000 358,000
Page 27 of 52
Personal copy of Barbin, Rafael R. (rafaelbarbin123@gmail.com)
Global Reciprocal Colleges,Inc-Caloocan

60. C.

1st Quarter 2nd 3rd 4th Quarter Itemized 8%


Quarter Quarter
Gross sales 500,000 600,000 700,000 800,000 2,600,00 2,600,00
0 0
Cost of sales 200,000 400,000 400,000 600,000 1,600,00 250,000
(purchases) 0
Gross income 300,000 200,000 300,000 200,000 1,000,00
0
Expenses 100,000 100,000 100,000 100,000 400,000
Net income 200,000 100,000 200,000 100,000 600,000 2,350,00
0
Income tax due 80,000 188,000

61. B. MCIT is payable.

1st Quarter
Gross sales 3,000,000
Cost of sales:
Beginning Inventory 500,000
Add: Purchases 3,500,000
Available for Sale 4,000,000
Less: Ending Inventory 2,000,000
Cost of Sales 2,000,000
Gross income
1,000,000
Expenses 2,000,000
Net income (loss) (1,000,000)

MCIT (1% of Gross income) 10,000


RCIT (25% of net income) 0

62. D.

1st Quarter Output tax Input tax


Gross sales 3,000,000 360,000
Purchases 3,500,000 420,000 (60,000)

63. C.

Itemized OSD
Gross sales 24,000,00 24,000,00
0 0
COS 16,000,00 16,000,00
0 0
Gross income 8,000,000 8,000,000
Deduction 7,000,000 3,200,000
Net income (loss) 1,000,000 4,800,000
RCIT (20%) 200,000 960,000
MCIT 160,000 160,000
Payable (RCIT) 200,000 960,000
Savings 760,000

Page 28 of 52
Personal copy of Barbin, Rafael R. (rafaelbarbin123@gmail.com)
Global Reciprocal Colleges,Inc-Caloocan

64. B.

1st Quarter 2nd Quarter 3rd Quarter 4th Quarter


Gross sales 3,000,000 6,000,000 7,000,000 8,000,000
Purchases 3,500,000 5,000,000 8,000,000 5,000,000
Output tax 360,000 720,000 840,000 960,000
Input tax 420,000 60,000* 960,000 120,000**
600,000 600,000
660,000 720,000
VAT Payable (60,000)* 60,000 (120,000)** 240,000

65. B.

1st Quarter 2nd Quarter 3rd Quarter 4th Quarter


Gross sales 3,000,000 6,000,000 7,000,000 8,000,000
Purchases 3,500,000 5,000,000 8,000,000 5,000,000
Output tax 360,000 720,000 840,000 960,000
Input tax 420,000 60,000* 960,000 120,000**
600,000 600,000
660,000 720,000
VAT Payable (60,000)* 60,000 (120,000)** 240,000

66. B.

Gross Receipts 40,000,000


Cost of Service:
Materials 20,000,000
Direct Labor (minimum wage earners) 10,000,000
Gross Income 10,000,000
Operating Expenses 15,000,000
Net Income (Loss) (5,000,000)

RCIT 0
MCIT (1% of gross income) 10,000

67. D.

Gross Receipts 40,000,000


Cost of Service:
Materials 20,000,000
Direct Labor (minimum wage earners) 10,000,000
Gross Income 10,000,000
Optional Standard Deduction 4,000,000
Net Income (Loss) 6,000,000

RCIT (25%) 1,500,000


MCIT (1% of gross income) 10,000

Page 29 of 52
Personal copy of Barbin, Rafael R. (rafaelbarbin123@gmail.com)
Global Reciprocal Colleges,Inc-Caloocan

68. C. Input taxes not declared in VAT returns are not creditable.

Collections per OR 30,000,000


Output tax 3,600,000
Less: input tax 1,920,000
Net VAT Payable 1,680,000
Less: tax paid 480,000
Deficiency Basic VAT 1,200,000

Alternative computation:
Collections per OR 30,000,000
Gross receipts per VAT 20,000,000
returns
Undeclared receipts 10,000,000
Deficiency Basic VAT 1,200,000

69. C
70. B

Page 30 of 52
Personal copy of Barbin, Rafael R. (rafaelbarbin123@gmail.com)
Global Reciprocal Colleges,Inc-Caloocan

Suggested Solution – RFBT

1. B. S1 – Citation of the law is RA 7394 or Consumer Protection Act. S2 – Premium pay is the amount given in
case a worker works on his rest day.
2. E. Wages shall be paid in cash, legal tender at or near the place of work (Art. 102 Labor Code).
3. C. E-Commerce Act gives validity and legal recognition to electronic documents, electronic signatures and
electronic transactions. It facilitates the admission of electronic documents and electronic signature as
evidence in cases of disputes.
4. B. S1 – Report is done within lemon law rights period i.e. ending twelve (12) months from the date of original
delivery to the consumer, or up to twenty thousand (20,000) kilometers of operation after such delivery,
whichever comes first. S2 – Plus 30% of the daily basic rate or a total of 230% for work performed on a
regular holiday falling on the employee’s rest day.
5. A.
6. B. All government agencies and local government units must post information billboards containing their most
current and updated service standards. This set of standards will be called the Citizen’s Charter.
7. C. S1 – Public monitoring allows public involvement of qualified and eligible Civil Society Organizations
(NGOs, PAs, academic Institutions and religious institutions) to observe and monitor the procurement process
until contract implementation. S2 – Statement is True.
8. D.
9. C. S1 – A worker below 15 should be directly under the sole responsibility of parents or guardians; in family
business; work does not interfere with the child’s schooling/normal development; with work permit from
DOLE, maximum hours 4 hours daily and 20 hours weekly only, no work from 8pm-6am (RA 9231). S2 –
Statement is True.
10. E. This is an agreement by two or more competing businesses to set or limit production levels and create an
artificial supply shortage, thereby raising prices. Similar forms of anti-competitive agreements include
restrictions in markets, technical development and investment.
11. C. S1 –In case of projects involving mixed procurements, the nature of the procurement, shall be determined
based on the primary purpose of the contract. S2 – Statement is True.
12. D. S1 – Statement is True. S2 – Direct Contracting, otherwise known as Single Source Procurement – a
method of Procurement that does not require elaborate Bidding Documents because the supplier is simply
asked to submit a price quotation or a pro-forma invoice together with the conditions of sale, which offer may
be accepted immediately or after some negotiations.
13. D. S1 – Statement is True. S2 – Vertical Relationship refers to a situation where an entity operates in a
market that is immediately upstream or downstream of a market in which another entity operates, such that
the two entities are in an actual or potential buyer-seller relationship.
14. D. S1 – Statement is True. S2 – It is not required. CBA is not required for the issuance of the Certificate of
Incorporation.
15. A. Not an exception.
16. B. S1 – 100% Rule - SC shall be distributed completely and equally to covered employees, except managers,
hence the 85%-15% Rule as been repealed (RA 11360, August 7, 2019). S2 – Nightshift differential
(NSD) pay: plus 10% of the basic/regular rate between 10 pm and 6 am.
17. D. S1 – Statement is True. S2 – Price discrimination is setting prices or terms that unreasonably exclude
some sellers or customers of the same goods or services.
18. B. The ownership of copyright belongs to the employee if not part of his regular duties and to the employer if
part of regular duties.
19. A. SEC. 12. Petition to Initiate Voluntary Proceedings by Debtor. — When approved by the owner in case of a
sole proprietorship, or by a majority of the partners in case of a partnership, or, in case of a corporation, by a
majority vote of the board of directors or trustees and authorized by the vote of the stockholders representing
at least two-thirds (2/3) of the outstanding capital stock, or in case of nonstock corporation, by the vote of at
least two-thirds (2/3) of the members, in a stockholder’s or member’s meeting duly called for the purpose, an
insolvent debtor may initiate voluntary proceedings under this Act by filing a petition for rehabilitation with
the court and on the grounds hereinafter specifically provided. (FRIA)
20. A. SEC. 4. Definition of Terms. — As used in this Act, the term: (w) Liquidator shall refer to the natural person
or juridical entity appointed as such by the court and entrusted with such powers and duties as set forth in
this Act: Provided, That, if the liquidator is a juridical entity, it must designate a natural person who possesses
all the qualifications and none of the disqualifications as its representative, it being understood that the
juridical entity and the representative are solidarily liable for all obligations and responsibilities of the
liquidator. (FRIA)
21. A.
22. B. SEC. 90. Voluntary Liquidation. — An insolvent debtor may apply for liquidation by filing a petition for
liquidation with the court. The petition shall be verified, shall establish the insolvency of the debtor and shall
contain, whether as an attachment or as part of the body of the petition:
(a) a schedule of the debtor’s debts and liabilities including a list of creditors with their addresses, amounts of
claims and collaterals, or securities, if any;
(b) an inventory of all its assets including receivables and claims against third parties; and
(c) the names of at least three (3) nominees to the position of liquidator.
23. B. SEC. 83. Out-of-Court or Informal Restructuring Agreements and Rehabilitation Plans. — An out-of-court or
informal restructuring agreement or Rehabilitation Plan that meets the minimum requirements prescribed in
this chapter is hereby recognized as consistent with the objectives of this Act.

Page 31 of 52
Personal copy of Barbin, Rafael R. (rafaelbarbin123@gmail.com)
Global Reciprocal Colleges,Inc-Caloocan

24. C. SEC. 105. Petition; Acts of Insolvency. — Any creditor or group of creditors with a claim of, or with claims
aggregating, at least Five hundred thousand pesos (Php500,000.00) may file a verified petition for liquidation
with the court of the province or city in which the individual debtor resides.

The following shall be considered acts of insolvency, and the petition for liquidation shall set forth or allege at
least one of such acts:
xxxx xxxx
(l) That for a period of thirty (30) days, he has failed, after demand, to pay any moneys deposited with him or
received by him in a fiduciary capacity;
25. C. It shall be the duty of the drawee of any check, when refusing to pay the same to the holder thereof upon
presentment, to cause to be written, printed, or stamped in plain language thereon, or attached thereto, the
reason for drawee's dishonor or refusal to pay the same: Provided, That where there are no sufficient funds in
or credit with such drawee bank, such fact shall always be explicitly stated in the notice of dishonor or refusal.
In all prosecutions under this Act, the introduction in evidence of any unpaid and dishonored check, having
the drawee's refusal to pay stamped or written thereon or attached thereto, with the reason therefore as
aforesaid, shall be prima facie evidence of the making or issuance of said check, and the due presentment to
the drawee for payment and the dishonor thereof, and that the same was properly dishonored for the reason
written, stamped or attached by the drawee on such dishonored check.
26. C. Loan function belongs to Banks under the General Banking Law.
27. A. Contracts executed between private persons and cooperatives prior to the registration of the cooperative
shall remain valid and binding between the parties and upon registration of the cooperative. (Art. 19, RA
9520)
28. A. Money market placements and trust funds are not covered. They are exclusions from the coverage of the
law.
29. A. Actual cases decided by GPPB
S1 - 2013-03-27, NPM 035-2013, Requesting Entity: Municipality of Hilongos, Issues Concern: Performance
Security and Blacklisting
S2 - 2013-04-10, NPM 036-2013, Requesting Entity: Municipality of Asipulo, Issues Concern: Marking of
Bids
30. A. All covered females, regardless of civil status, employment status and legitimacy of her child shall be
granted maternity leave. (IRR, RA 112110) To be eligible for maternity benefit from the Social Security
System (SSS), an employee must have paid at least 3 monthly SSS contributions in the 12 month period
immediately preceding the semester of the birth, miscarriage or termination.
31. D. Bank deposits may be garnished by creditors of the depositor, and it will not be considered as a violation of
the Bank Secrecy Law. The reason is that the amount of deposit is actually not disclosed.
32. C. As amended by RA 10365, Sec. 6 (a) provides that any person may be charged with and convicted of both
the offense of money laundering and unlawful activity. It further states that the prosecution of any offense or
violation under the law shall proceed independently of any proceeding relating to the unlawful activity.
33. A. The following categories of credit transactions are outside the scope of these regulations:
(1) Credit transactions which do not involve the payment of any finance charge by the debtor; and
(2) Credit transactions in which the debtor is the one specifying a definite and fixed set of credit terms such
as bank deposits, insurance contracts, sale of bonds, etc.
34. B. BOD must authorize the person to represent the corporation or must at least be authorized by the By-laws
of the Corporation.
35. E. SEC. 32. Contracts Between Corporations with Interlocking Directors. – Except in cases of fraud, and
provided the contract is fair and reasonable under the circumstances, a contract between two (2) or more
corporations having interlocking directors shall not be invalidated on that ground alone: Provided, That if the
interest of the interlocking director in one (1) corporation is substantial and the interest in the other
corporation or corporations is merely nominal, the contract shall be subject to the provisions of the preceding
section insofar as the latter corporation or corporations are concerned. Stockholdings exceeding twenty
percent (20%) of the outstanding capital stock shall be considered substantial for purposes of interlocking
directors.
36. B. SEC. 32. Contracts Between Corporations with Interlocking Directors. – Except in cases of fraud, and
provided the contract is fair and reasonable under the circumstances, a contract between two (2) or more
corporations having interlocking directors shall not be invalidated on that ground alone: Provided, That if the
interest of the interlocking director in one (1) corporation is substantial and the interest in the other
corporation or corporations is merely nominal, the contract shall be subject to the provisions of the preceding
section – [SEC. 31. Dealings of Directors, Trustees or Officers with the Corporation. – A contract of the
corporation with (1) one or more of its directors, trustees, officers or their spouses and relatives within the
fourth civil degree of consanguinity or affinity is voidable, at the option of such corporation] insofar as the
latter corporation or corporations are concerned. Stockholdings exceeding twenty percent (20%) of the
outstanding capital stock shall be considered substantial for purposes of interlocking directors.
37. C. The Securities and Exchange Commission (SEC) has tightened the rules governing the disqualification and
removal of directors, trustees, and officers of corporations, in line with its commitment to ensure good
corporate governance in the country.

Page 32 of 52
Personal copy of Barbin, Rafael R. (rafaelbarbin123@gmail.com)
Global Reciprocal Colleges,Inc-Caloocan

38. A. SEC. 97. Validity of Restrictions on Transfer of Shares. – Restrictions on the right to transfer shares must
appear in the articles of incorporation, in the bylaws, as well as in the certificate of stock; otherwise, the same
shall not be binding on any purchaser in good faith. Said restrictions shall not be more onerous than granting
the existing stockholders or the corporation the option to purchase the shares of the transferring stockholder
with such reasonable terms, conditions or period stated. If, upon the expiration of said period, the existing
stockholders or the corporation fails to exercise the option to purchase, the transferring stockholder may sell
their shares to any third person.
39. A. Doctrine of Corporate Fiction i.e. separate and distinct personality
40. D. SEC. 66. Payment of Balance of Subscription. – Subject to the provisions of the subscription contract, the
board of directors may, at any time, declare due and payable to the corporation unpaid subscriptions and may
collect the same or such percentage thereof, in either case, with accrued interest, if any, as it may deem
necessary. Payment of unpaid subscription or any percentage thereof, together with any interest accrued shall
be made on the date specified in the subscription contract or on the date stated in the call made by the board.
Failure to pay on such date shall render the entire balance due and payable and shall make the stockholder
liable for interest at the legal rate on such balance, unless a different interest rate is provided in the
subscription contract. The interest shall be computed from the date specified, until full payment of the
subscription. If no payment is made within thirty (30) days from the said date, all stocks covered by
the subscription shall thereupon become delinquent and shall be subject to sale as hereinafter provided,
unless the board of directors orders otherwise.
41. D. SEC. 37. Power to Increase or Decrease Capital Stock; Incur, Create or Increase Bonded Indebtedness. –
No corporation shall increase or decrease its capital stock or incur, create or increase any bonded
indebtedness UNLESS APPROVED BY A MAJORITY VOTE OF THE BOARD OF DIRECTORS AND BY two-thirds
(2/3) of the outstanding capital stock at a stockholders’ meeting duly called for the purpose. Written notice of
the time and place of the stockholders’ meeting and the purpose for said meeting must be sent to the
stockholders at their places of residence as shown in the books of the corporation and served on the
stockholders personally, or through electronic means recognized in the corporation’s bylaws and/or the
Commission’s rules as a valid mode for service of notices.
42. A. Principle of Non-Formalities - “Creation equals Protection”
43. A. SEC. 71. Rights of Unpaid Shares, Nondelinquent. – Holders of subscribed shares not fully paid which are
not delinquent shall have all the rights of a stockholder.
44. D. SEC. 8. Redeemable Shares. – Redeemable shares may be issued by the corporation when expressly
provided in the articles of incorporation. They are shares which may be purchased by the corporation from the
holders of such shares upon the expiration of a fixed period, regardless of the existence of unrestricted
retained earnings in the books of the corporation, and upon such other terms and conditions stated in the
articles of incorporation and the certificate of stock representing the shares, subject to rules and regulations
issued by the Commission.
45. C. SEC. 20. Corporation by Estoppel. – All persons who assume to act as a corporation knowing it to be
without authority to do so shall be liable as general partners for all debts, liabilities and damages incurred or
arising as a result thereof: Provided, however, That when any such ostensible corporation is sued on any
transaction entered by it as a corporation or on any tort committed by it as such, it shall not be allowed to use
its lack of corporate personality as a defense. Anyone who assumes an obligation to an ostensible corporation
as such cannot resist performance thereof on the ground that there was in fact no corporation.
46. A. Any individual or corporation can be insured, provided the said individual or corporation is not a public
enemy. A public enemy refers to a citizen of an enemy state.
47. B. It is a provision in law that after a policy of life insurance made payable on the death of the insured shall
have been in force during the lifetime of the insured for a period of two (2) years from the date of its issue or
of is last reinforcement, the insurer cannot prove that the policy is void ab initio or is rescindible by reason of
fraudulent concealment or misrepresentations of the insured or his agent.
48. A. An insurance endorsement (or rider) modifies an insurance policy to either add, delete, or exclude certain
types of coverage. One of the most common ways owners use endorsements is to add additional insurance
protection to a policy. This might be because the policy specifically excludes it or because it simply doesn’t
mention the risk at all.
49. B. Beneficiary is the third party in whose favor a stipulation is made. One who receives a benefit or
advantage, or who is entitled to the benefit of the contract, that is, one to whom the insurance is payable or
who is entitled to the proceeds of the policy on the occurrence of the event designated.
50. A. A De jure corporation is a corporation which is existing in fact and in law.
51. C. Doctrine of Corporate Fiction i.e. separate and distinct personality.
52. A. SEC. 63. Issuance of Stock Certificates. – No certificate of stock shall be issued to a subscriber until the full
amount of the subscription together with interest and expenses (in case of delinquent shares), if any is due,
has been paid.
53. B. SEC. 78. Effectivity of Merger or Consolidation. – The articles of merger or of consolidation, signed and
certified as required by this Code, shall be submitted to the Commission for its approval: Provided, That in the
case of merger or consolidation of banks or banking institutions, loan associations, trust companies, insurance
companies, public utilities, educational institutions, and other special corporations governed by special laws,
the favorable recommendation of the appropriate government agency shall first be obtained. If the
Commission is satisfied that the merger or consolidation of the corporations concerned is consistent with the
provisions of this Code and existing laws, it shall issue a certificate approving the articles and plan of
merger or of consolidation, at which time the merger or consolidation shall be effective.

Page 33 of 52
Personal copy of Barbin, Rafael R. (rafaelbarbin123@gmail.com)
Global Reciprocal Colleges,Inc-Caloocan

54. D. SEC. 150. Doing Business Without a License. – No foreign corporation transacting business in the
Philippines without a license, or its successors or assigns, shall be permitted to maintain or intervene in any
action, suit or proceeding in any court or administrative agency of the Philippines; but such corporation may
be sued or proceeded against before Philippine courts or administrative tribunals on any valid cause of action
recognized under Philippine laws.
55. B. A shareholder derivative suit is a legal action filed by an individual shareholder, in the name of the
company, to redress wrongs or harms to the company.
56. A. SEC. 151. Revocation of License. – Without prejudice to other grounds provided under special laws, the
license of a foreign corporation to transact business in the Philippines may be revoked or suspended by the
Commission upon any of the following grounds:(b) Failure to appoint and maintain a resident agent in the
Philippines as required by this Title;
57. A. As a general rule, a bank has a right of set off of the deposits in its hands for the payment of any
indebtedness to it on the part of a depositor. [Gullas v. PNB (1935)]
58. A. The Ombudsman has the power to issue subpoena and subpoena duces tecum, take testimony in any
investigation or inquiry, as well as examine and access bank accounts and records. Bank deposits of a public
official, his spouse and unmarried children may be taken into consideration in the enforcement of Section 8 of
The Anti-Graft and Corrupt Practices Act.
59. A. The Chattel Mortgage Law, in its section 5, in describing what shall be deemed sufficient to constitute a
good chattel mortgage, includes the requirement of an affidavit of good faith appended to the mortgage and
recorded therewith.
60. A. Deficiency judgment is allowed in real mortgage. BUT not allowed in pledge, Recto Law and third-party
mortgagor.
61. B. TILA law requires full disclosure by requiring the lender to give the borrower all the details regarding the
transaction. It is called “Material Disclosures.”
62. C. The object of a notice of sale is to inform the public of the nature and condition of the property to be sold,
and of the time, place and terms of the sale. Notices are given for the purpose of securing bidders and to
prevent a sacrifice [sale] of the property. The goal of the notice requirement is to achieve a "reasonably wide
publicity" of the auction sale. This is why publication in a newspaper of general circulation is required. The
Court has previously taken judicial notice of the "far-reaching effects" of publishing the notice of sale in a
newspaper of general circulation. While the law recognizes the right of a bank to foreclose a mortgage upon
the mortgagor’s failure to pay his obligation, it is imperative that such right be exercised according to its clear
mandate. Each and every requirement of the law must be complied with, lest, the valid exercise of the right
would end. It must be remembered that the exercise of a right ends when the right disappears, and it
disappears when it is abused especially to the prejudice of others. Consequently, the questioned extrajudicial
foreclosure of real estate mortgage and sale are void. (Phil. Savings Bank v. Geronimo, April 19, 2010)
63. C. Respondeat superior embodies the general rule that an employer is responsible for the negligent acts or
omissions of its employees.
64. C. SEC. 23. Election of Directors or Trustees. – Except when the exclusive right is reserved for holders of
founders’ shares under Section 7 of this Code, each stockholder or member shall have the right to nominate
any director or trustee who possesses all of the qualifications and none of the disqualifications set forth in this
Code.
xxxx xxx
Unless otherwise provided in the articles of incorporation or in the bylaws, members of nonstock
corporations may cast as many votes as there are trustees to be elected but may not cast more
than one (1) vote for one (1) candidate.
SEC. 88. Right to Vote. – The right of the members of any class or classes to vote may be limited, broadened,
or denied to the extent specified in the articles of incorporation or the bylaws. Unless so limited, broadened,
or denied, each member, regardless of class, shall be entitled to one (1) vote. Unless otherwise provided in
the articles of incorporation or the bylaws, a member may vote by proxy, in accordance with the provisions of
this Code. The bylaws may likewise authorize voting through remote communication and/or in absentia.
65. A. SEC. 81. How Right is Exercised. – The dissenting stockholder who votes against a proposed corporate
action may exercise the right of appraisal by making a written demand on the corporation for the payment of
the fair value of shares held within thirty (30) days from the date on which the vote was taken: Provided,
That failure to make the demand within such period shall be deemed a waiver of the appraisal right. If the
proposed corporate action is implemented, the corporation shall pay the stockholder, upon surrender of the
certificate or certificates of stock representing the stockholder’s shares, the fair value thereof as of the
day before the vote was taken, excluding any appreciation or depreciation in anticipation of such
corporate action.
66. B. SEC. 22. The Board of Directors or Trustees of a Corporation; Qualification and Term. – Unless otherwise
provided in this Code, the board of directors or trustees shall exercise the corporate powers, conduct all
business, and control all properties of the corporation. Directors shall be elected for a term of one (1) year
from among the holders of stocks registered in the corporation’s books, while trustees shall be elected for a
term not exceeding three (3) years from among the members of the corporation. Each director and trustee
shall hold office until the successor is elected and qualified. A director who ceases to own at least one (1)
share of stock or a trustee who ceases to be a member of the corporation shall cease to be such.

SEC. 28. Vacancies in the Office of Director or Trustee; Emergency Board. –


xxxx xxx
A director or trustee elected to fill a vacancy shall be referred to as replacement director or trustee and shall
serve only for the unexpired term of the predecessor in office.

Page 34 of 52
Personal copy of Barbin, Rafael R. (rafaelbarbin123@gmail.com)
Global Reciprocal Colleges,Inc-Caloocan

67. C. SEC. 41. Power to Invest Corporate Funds in Another Corporation or Business or for Any Other Purpose. –
Subject to the provisions of this Code, a private corporation may invest its funds in any other corporation,
business, or for any purpose other than the primary purpose for which it was organized, when approved by
a majority of the board of directors or trustees and ratified by the stockholders representing at
least two-thirds (2/3) of the outstanding capital stock, or by at least two thirds (2/3) of the members
in the case of nonstock corporations, at a meeting duly called for the purpose. Notice of the proposed
investment and the time and place of the meeting shall be addressed to each stockholder or member at the
place of residence as shown in the books of the corporation and deposited to the addressee in the post office
with postage prepaid, served personally, or sent electronically in accordance with the rules and regulations of
the Commission on the use of electronic data message, when allowed by the bylaws or done with the consent
of the stockholders: Provided, That any dissenting stockholder shall have appraisal right as provided in this
Code: Provided, however, That where the investment by the corporation is reasonably necessary to
accomplish its primary purpose as stated in the articles of incorporation, the approval of the stockholders or
members shall not be necessary.

SEC. 44. Ultra Vires Acts of Corporations. – No corporation shall possess or exercise corporate powers other
than those conferred by this Code or by its articles of incorporation and except as necessary or incidental to
the exercise of the powers conferred.

An illegal act is void and cannot be validated while an ultra vires act is merely voidable and can be
enforced by performance, ratification or estoppel, or on equitable grounds.
68. A. SEC. 81. How Right is Exercised. –
xxx xxx
If, within sixty (60) days from the approval of the corporate action by the stockholders, the withdrawing
stockholder and the corporation cannot agree on the fair value of the shares, it shall be determined and
appraised by three (3) disinterested persons, one of whom shall be named by the stockholder, another by
the corporation, and the third by the two (2) thus chosen. The findings of the majority of the appraisers shall
be final, and their award shall be paid by the corporation within thirty (30) days after such award is made:
Provided, That no payment shall be made to any dissenting stockholder unless the corporation has
unrestricted retained earnings in its books to cover such payment: Provided, further, That upon payment by
the corporation of the agreed or awarded price, the stockholder shall forthwith transfer the shares to the
corporation.
69. D. No deficiency judgment against a third-party mortgagor. And the liability of the third-party mortgagors
extends only to the property mortgaged. Should there be any deficiency, the creditors has recourse on the
principal debtor. (Cerna v. CA, March 30, 1993)
70. D. SEC. 36. Power to Extend or Shorten Corporate Term. – A private corporation may extend or shorten its
term as stated in the articles of incorporation when approved by a majority vote of the board of directors or
trustees, and ratified at a meeting by the stockholders or members representing at least two-thirds
(2/3) of the outstanding capital stock or of its members. Written notice of the proposed action and the
time and place of the meeting shall be sent to stockholders or members at their respective place of residence
as shown in the books of the corporation, and must either be deposited to the addressee in the post office
with postage prepaid, served personally, or when allowed in the bylaws or done with the consent of the
stockholder, sent electronically in accordance with the rules and regulations of the Commission on the use of
electronic data messages. In case of extension of corporate term, a dissenting stockholder may exercise the
right of appraisal under the conditions provided in this Code.
71. D. A partner whose membership in a partnership is kept secret from the public. - Merriam and Webster
72. C. ARTICLE 1795. The risk of specific and determinate things, which are not fungible, contributed to the
partnership so that only their use and fruits may be for the common benefit, shall be borne by the partner
who owns them.
If the things contribute are fungible, or cannot be kept without deteriorating, or if they were contributed to be
sold, the risk shall be borne by the partnership. In the absence of stipulation, the risk of things brought and
appraised in the inventory, shall also be borne by the partnership, and in such case the claim shall be limited
to the value at which they were appraised. (1687)
73. C. ARTICLE 1769. In determining whether a partnership exists, these rules shall apply:
xxxx xxx

(4) The receipt by a person of a share of the profits of a business is prima facie evidence that he is a partner
in the business, but no such inference shall be drawn if such profits were received in payment:
74. B. ARTICLE 1810. The property rights of a partner are:
(1) His rights in specific partnership property;
(2) His interest in the partnership; and
(3) His right to participate in the management. (n)

Page 35 of 52
Personal copy of Barbin, Rafael R. (rafaelbarbin123@gmail.com)
Global Reciprocal Colleges,Inc-Caloocan

75. B. Article 1825. When a person, by words spoken or written or by conduct, represents himself, or consents to
another representing him to anyone, as a partner in an existing partnership or with one or more persons not
actual partners, he is liable to any such persons to whom such representation has been made, who has, on
the faith of such representation, given credit to the actual or apparent partnership, and if he has made such
representation or consented to its being made in a public manner he is liable to such person, whether the
representation has or has not been made or communicated to such person so giving credit by or with the
knowledge of the apparent partner making the representation or consenting to its being made: (1) When a
partnership liability results, he is liable as though he were an actual member of the partnership; (2) When no
partnership liability results, he is liable pro rata with the other persons, if any, so consenting to the contract or
representation as to incur liability, otherwise separately. When a person has been thus represented to be a
partner in an existing partnership, or with one or more persons not actual partners, he is an agent of the
persons consenting to such representation to bind them to the same extent and in the same manner as
though he were a partner in fact, with respect to persons who rely upon the representation. When all the
members of the existing partnership consent to the representation, a partnership act or obligation results; but
in all other cases it is the joint act or obligation of the person acting and the persons consenting to the
representation. (n)
76. B. Husband and wife are not allowed to enter into universal partnership. But are allowed to enter into
particular and limited partnership.
77. A. Article 1616. The vendor cannot avail himself of the right of repurchase without returning to the vendee the
price of the sale, and in addition:
(1) The expenses of the contract, and any other legitimate payments made by reason of the sale;
(2) The necessary and useful expenses made on the thing sold. (1518)
78. C. Article 1537. The vendor is bound to deliver the thing sold and its accessions and accessories in the
condition in which they were upon the perfection of the contract.
All the fruits shall pertain to the vendee from the day on which the contract was perfected. (1468a)
79. D. Article 1526. Subject to the provisions of this Title, notwithstanding that the ownership in the goods may
have passed to the buyer, the unpaid seller of goods, as such, has:
(1) A lien on the goods or right to retain them for the price while he is in possession of them; (2) In case of
the insolvency of the buyer, a right of stopping the goods in transitu after he has parted with the possession
of them;
(3) A right of resale as limited by this Title;
(4) A right to rescind the sale as likewise limited by this Title.
80. A. See Art 1171 NCC.
81. A. Article 1339. Failure to disclose facts, when there is a duty to reveal them, as when the parties are bound
by confidential relations, constitutes fraud. (n)
82. D. Article 1180. When the debtor binds himself to pay when his means permit him to do so, the obligation
shall be deemed to be one with a period, subject to the provisions of article 1197. (n)
83. A. Article 1198. The debtor shall lose every right to make use of the period: (1) When after the obligation has
been contracted, he becomes insolvent, unless he gives a guaranty or security for the debt; (2) When he does
not furnish to the creditor the guaranties or securities which he has promised; (3) When by his own acts he
has impaired said guaranties or securities after their establishment, and when through a fortuitous event they
disappear, unless he immediately gives new ones equally satisfactory; (4) When the debtor violates any
undertaking, in consideration of which the creditor agreed to the period; (5) When the debtor attempts to
abscond. (1129A)
84. A. Article 1211. Solidarity may exist although the creditors and the debtors may not be bound in the same
manner and by the same periods and conditions. (1140)
85. A. Article 1245. Dation in payment, whereby property is alienated to the creditor in satisfaction of a debt in
money, shall be governed by the law of sales. (n)
86. B. Article 1359. When, there having been a meeting of the minds of the parties to a contract, their true
intention is not expressed in the instrument purporting to embody the agreement, by reason of mistake,
fraud, inequitable conduct or accident, one of the parties may ask for the reformation of the instrument to the
end that such true intention may be expressed. If mistake, fraud, inequitable conduct, or accident has
prevented a meeting of the minds of the parties, the proper remedy is not reformation of the instrument but
annulment of the contract.
87. D. Statute of Frauds applies only to executory contracts. Contract of sale has been executed (delivered and
paid).
88. B. Article 1279. In order that compensation may be proper, it is necessary: (1) That each one of the obligors
be bound principally, and that he be at the same time a principal creditor of the other; (2) That both debts
consist in a sum of money, or if the things due are consumable, they be of the same kind, and also of the
same quality if the latter has been stated; (3) That the two debts be due; (4) That they be liquidated and
demandable; (5) That over neither of them there be any retention or controversy, commenced by third
persons and communicated in due time to the debtor. (1196)
89. B. Cognition theory is followed in the Philippine jurisdiction (See Art 1319, 2nd par.)

Page 36 of 52
Personal copy of Barbin, Rafael R. (rafaelbarbin123@gmail.com)
Global Reciprocal Colleges,Inc-Caloocan

90. D. SEC. 47. Amendment to Bylaws. – A majority of the board of directors or trustees, and the owners
of at least a majority of the outstanding capital stock, or at least a majority of the members of a
nonstock corporation, at a regular or special meeting duly called for the purpose, may amend or
repeal the bylaws or adopt new bylaws. The owners of two-thirds (2/3) of the outstanding capital stock
or two-thirds (2/3) of the members in a nonstock corporation may delegate to the board of directors or
trustees the power to amend or repeal the bylaws or adopt new bylaws: Provided, That any power delegated
to the board of directors or trustees to amend or repeal the bylaws or adopt new bylaws shall be considered
as revoked whenever stockholders owning or representing a majority of the outstanding capital stock or
majority of the members shall so vote at a regular or special meeting. Whenever the bylaws are amended or
new bylaws are adopted, the corporation shall file with the Commission such amended or new bylaws and, if
applicable, the stockholders’ or members’ resolution authorizing the delegation of the power to amend and/or
adopt new bylaws, duly certified under oath by the corporate secretary and a majority of the directors or
trustees. The amended or new bylaws shall only be effective upon the issuance by the Commission of a
certification that the same is in accordance with this Code and other relevant laws.
91. D. SEC. 82. Effect of Demand and Termination of Right. – From the time of demand for payment of the fair
value of a stockholder’s shares until either the abandonment of the corporate action involved or the purchase
of the said shares by the corporation, all rights accruing to such shares, including voting and dividend rights,
shall be suspended in accordance with the provisions of this Code, except the right of such stockholder to
receive payment of the fair value thereof: Provided, That if the dissenting stockholder is not paid the
value of the said shares within thirty (30) days after the award, the voting and dividend rights shall
immediately be restored
92. B. SEC. 42. Power to Declare Dividends. – The board of directors of a stock corporation may declare dividends
out of the unrestricted retained earnings which shall be payable in cash, property, or in stock to all
stockholders on the basis of outstanding stock held by them: Provided, That any cash dividends due on
delinquent stock shall first be applied to the unpaid balance on the subscription plus costs and
expenses, while stock dividends shall be withheld from the delinquent stockholders until their
unpaid subscription is fully paid: Provided, further, That no stock dividend shall be issued without the
approval of stockholders representing at least two-thirds (2/3) of the outstanding capital stock at a regular or
special meeting duly called for the purpose. Stock corporations are prohibited from retaining surplus profits in
excess of one hundred percent (100%) of their paid-in capital stock, except: (a) when justified by definite
corporate expansion projects or programs approved by the board of directors; or (b) when the corporation is
prohibited under any loan agreement with financial institutions or creditors, whether local or foreign, from
declaring dividends without their consent, and such consent has not yet been secured; or (c) when it can be
clearly shown that such retention is necessary under special circumstances obtaining in the corporation, such
as when there is need for special reserve for probable contingencies.
93. D. SEC. 15. Amendment of Articles of Incorporation. – Unless otherwise prescribed by this Code or by special
law, and for legitimate purposes, any provision or matter stated in the articles of incorporation may be
amended by a majority vote of the board of directors or trustees and the vote or written assent of the
stockholders representing at least two-thirds (2/3) of the outstanding capital stock, without prejudice to the
appraisal right of dissenting stockholders in accordance with the provisions of this Code. The articles of
incorporation of a nonstock corporation may be amended by the vote or written assent of majority of the
trustees and at least two-thirds (2/3) of the members. The original and amended articles together shall
contain all provisions required by law to be set out in the articles of incorporation. Amendments to the articles
shall be indicated by underscoring the change or changes made, and a copy thereof duly certified under oath
by the corporate secretary and a majority of the directors or trustees, with a statement that the amendments
have been duly approved by the required vote of the stockholders or members, shall be submitted to the
Commission. The amendments shall take effect upon their approval by the Commission or from the date of
filing with the said Commission if not acted upon within six (6) months from the date of filing for a
cause not attributable to the corporation.
94. A. The amendments shall take effect upon its approval by the authority OR within thirty (30) days from
the date of filing thereof if not acted upon by the Authority for a cause not attributable to the
cooperative. (Art 18, RA 9520)
95. C. The name of the incorporators is one of permanent character. Once an incorporator, forever an
incorporator. (See Sec 15, RA 11232)
96. C. Art. 35. Quorum. –A quorum shall consist of AT LEAST twenty-five per centum (25%) of all the members
entitled to vote (GR).
97. A. Art. 21. Two (2) or more cooperatives may merge into a single cooperative which shall either be one of the
constituent cooperatives or the consolidated cooperative.
No merger or consolidation shall be valid unless approved by a three-fourths (3/4) vote of all the
members with voting rights, present and constituting a quorum of each of the constituent cooperatives at
separate general assembly meetings.
The dissenting members shall have the right to exercise their right to withdraw their membership.
98. D. See RA 11521, January 30, 2021.
99. D. Sec. 16, A.M. No. 11-05-04-SC.
100. A.

Page 37 of 52
Personal copy of Barbin, Rafael R. (rafaelbarbin123@gmail.com)
Global Reciprocal Colleges,Inc-Caloocan

Suggested Solution – FAR

Question No. 1 - D
Question No. 2 - C
Question No. 3 - D
Question No. 4 - D
Question No. 5 - B
Purchase price (P130,000 -P10,000) 120,000
Transport cost to retail outlet 5,000
Total cost 125,000

Note: Transport cost to customers and other selling costs are recognized as expenses.

Question No. 6 - A
Total cost of units available for
sale in July 118,800
Less cost of goods sold during July 99,000
Inventory, July 31 19,800

Question No. 7 - C
Cost of materials (P78,400 - P8,400) 70,000
Labor costs 43,000
Allocated overhead costs 22,000
Allocated interest costs 10,000
Costs of installation 12,000
Safety inspection costs 4,000
Total cost of machine 161,000

Question No. 8 - A
Carrying amount, 8/1/21 200,000
Carrying amount of asset sold (P25,000 + P5,000) (30,000)
Depreciation for the year (20,000)
Carrying amount, 7/31/22 150,000

Question No. 9 - C
Land Building
Fair value 8,100,000 39,000,000
Less carrying amount 6,000,000 33,750,000
Revaluation increase (decrease) 2,100,000 5,250,000

Revaluation surplus - land 2,100,000


Revaluation surplus - building 5,250,000
Revaluation surplus, 1/1/22 7,350,000
Realized in 2022 (P5,250,000/30) (175,000)
Revaluation surplus, 12/31/22 7,175,000

Notes:
1. The building is 25% depreciated as of Jan. 1, 2022 (P11,250,000/P45,000,000)
2. Useful life of building is 40 years (10/.25)
3. Remaining useful life of building as of Jan. 1, 2022 is 30 years (40 - 10)

Question No. 10 - D
Depreciation (P4,000,000/20 x 6/12) (100,000)
Revaluation loss:
Fair value, 7/1/22 3,780,000
Less CA, 7/1/22 (P4,000,000 - P100,000) 3,900,000 (120,000)
Fair value adjustment gain:
Fair value, 12/31/22 3,850,000
Less fair value, 7/1/22 3,780,000 70,000
Net amount in profit or loss (150,000)

Page 38 of 52
Personal copy of Barbin, Rafael R. (rafaelbarbin123@gmail.com)
Global Reciprocal Colleges,Inc-Caloocan

Question No. 11 - C
Cost of brand 900,000
Amortization - 2022 [(P900,000/25) x (5/12)] (15,000)
Carrying amount, 12/31/22 885,000

Question No. 12 - A
Acquisition cost 10,000,000
Exploration and development costs 4,000,000
Residual value (2,000,000)
Amount subject to depletion 12,000,000
/Estimated reserves 35,000
Depletion rate 342.86

Depletion - 2022 (2,800 x P342.86) 960,008

Question No. 13 - C
Recoverable amount, 12/31/21 80,000
Carrying amount, 12/31/21 100,000
Impairment gain (loss) (20,000)

Recoverable amount, 12/31/22 84,000


Carrying amount, 12/31/22 80,000
Impairment gain (loss) 4,000

Question No. 14 - C
Impairment loss on Oct. 1:
Carrying amount after revaluation 595,000

Fair value less costs to sell (P595,000 - P25,000) 570,000 25,000


Impairment loss on Dec. 31:
Carrying amount 570,000

Fair value less costs to sell (P588,000 - P28,000) 560,000 10,000


Total impairment loss in profit or loss 35,000

Notes:
1) The revaluation decrease of P5,000 (P595,0000 - P600,000) on Oct. 1 is debited to revalution surplus.
2) Any impairment loss that arises by using the measurement principles in PFRS 5 must be
recognized in profit or loss even for an asset previously carried at a revalued amount.

Question No. 15 - B
Depreciation for 2022 [(P60M - P50M)/25] 400,000
Question No. 16 - B
Weighted average expenditures 8,500,000
Specific borrowing (6,000,000)
General borrowing 2,500,000
Capitalization rate - general (see below) 0.06
Capitalizable borrowing costs - general 150,000
Capitalizable borrowing costs - specific (P6,000,000 x .07) 420,000
Total capitalizable borrowing costs 570,000

Interest on 5% bonds (P10,000,000 x .05) 500,000


Interest on 8% notes (P5,000,000 x .08) 400,000
Total 900,000
/ General borrowings 15,000,000
Capitalization rate - general 0.06

Page 39 of 52
Personal copy of Barbin, Rafael R. (rafaelbarbin123@gmail.com)
Global Reciprocal Colleges,Inc-Caloocan

Question No. 17 - A
Petty cash (P500 - P100) 400
Cash in bank:
Balance per bank 25,500
Outstanding check (2,500)
Undeposited receipts (P12,000 - P500) 11,500
Currencies and coins awaiting deposit 5,500 40,000
Total cash 40,400

Question No. 18 - B
Bills and coins on hand 52,780
Petty cash (P1,000 - P650) 350
Money order 800
Checking account balance in Bank of P.I. 22,000
Correct cash balance 75,930

Question No. 19 - A
Unadjusted bank balance, 12/31 106,000
Deposits in transit (g) 24,000
Outstanding checks (h) (35,000)
Erroneous bank credit (i) (20,000)
Adjusted bank balance, 12/31 75,000

Note: This is the same as the adjusted book balance.

Question No. 20 - B
Accounts receivable at 12/31/2021 1,300,000
Credit sales 5,400,000
Collections from customers (4,750,000)
Accounts written off (125,000)
Collection of accounts written off in prior year -
Accounts receivable at 12/31/2022 1,825,000
Estimated uncollectible receivables per aging (165,000)
Amortized cost, 12/31/22 1,660,000

Question No. 21 - A
Accounts receivable, 1/1 800,000
Accounts receivable collected (2,600,000)
Credit sales:
Inventory, Jan. 1 1,200,000
Purchases 2,000,000
Inventory, Dec. 31 (1,100,000)
Cost of sales 2,100,000
Gross profit on sales 900,000
Total sales 3,000,000
Cash sales (500,000) 2,500,000
Accounts receivable, 12/31 700,000

Question No. 22 - B
Principal 20,000
Interest for 2023 (P20,000 x .08) 1,600
Total remaining cash flows, 12/31/22 21,600
PVF at 15%, 1 period 0.8696
Carrying amount, 12/31/22 18,783

Page 40 of 52
Personal copy of Barbin, Rafael R. (rafaelbarbin123@gmail.com)
Global Reciprocal Colleges,Inc-Caloocan

Question no. 23 - C
Present value of principal (P1,000,000 x 0.7118) 711,800
Present value of interest (P1,000,000 x 10% x 2.4018) 240,180
Gross CA, 12/31/22 951,980
Present value of expected cash flows:
Reduced principal (P600,000 x 0.7118) 427,080
Interest (P1,000,000 x 10% x 2.4018) 240,180 667,260
Loss allowance, 12/31/22 284,720

Question no. 24 - C
Principal 40,000
Interest to maturity (P40,000 x .06) 2,400
Maturity value 42,400
Discount (P42,400 x .1 x 9/12) (3,180)
Proceeds 39,220

Question no. 25 - D
PVF at 8.08%,
Cash flows 2 periods CA, 12/31/22
Principal + Redemption premium 11,000,000 0.8561 9,417,100
Interest 500,000 1.7813 890,650
10,307,750

The effective interest rate is 8.08% computed through interpolation. To


check:
PVF at 8.08%,
Cash flows 3 periods CA, 1/1/22
Principal + Redemption premium 11,000,000 0.7921 8,713,100
Interest 500,000 2.5734 1,286,700
9,999,800

Difference of P200 due to rounding.

Question No. 26 - C
The effective interest rate is 8% computed through interpolation. To check:
PVF at 8%,
Cash flows 3 periods CA, 1/1/21
Principal 1,000,000 0.7938 793,800
Interest 100,000 2.5771 257,710
1,051,510

Sales proceeds (P1M x


1.05) 1,050,000
CA, 12/31/22 (P1.1M x 0.9259) (1,018,490)
Gain on sale of bonds 31,510

Question No. 27 - D
Sales proceeds - Security
A 100,000
CA of investment sold (P140,000 x 1/2) (70,000)
Gain of sale of investment 30,000
Fair value adjustment on remaining
FVTPL:
Fair value, 12/31/21 (P90,000 + P110,000) 200,000
Less CA before FV adj. ([P140,000 x 1/2) + P100,000] 170,000 30,000
Total amount in 2022 profit or loss 60,000

Note: For FA at FVTOCI, amounts presented in OCI shall not be subsequently transferred to P/L.
However, the entity may transfer the cumulative gain or loss within equity. (PFRS 9 B5.7.1)

Page 41 of 52
Personal copy of Barbin, Rafael R. (rafaelbarbin123@gmail.com)
Global Reciprocal Colleges,Inc-Caloocan

Question No. 28 - A
Purchase price (Cost) 1,500,000
Share of profit of
associate:
Share on reported profit (P600,000 x .3) 180,000
Amortization of excess-equipment [(P1M x .3)/10] (30,000)
Amortization of excess-license [(P1M x .3)/10] (15,000) 135,000
Dividends received (50,000)
Carrying amount,
12/31/22 1,585,000

Question No. 29 - C
Annual premium 31,800
Increase in CSV during 2022 (P170,100 - P165,000) (5,100)
Dividends received (3,300)
Unexpired insurance premium refunded (P31,800 x 1/2) (15,900)
Life insurance expense 7,500

Question No. 30 - D
Question No. 31 - B
Question No. 32 - A
Question No. 33 - D

Unadjusted accounts payable 200,000


Debit balance in accounts payable 50,000
Post date checks issued 25,000
Adjusted accounts payable 275,000

Question No. 34 - B
Carrying amount 5,000,000
Less PV of modified cash flows:
Principal (P4,000,000 x 0.6575) 2,630,000
Interest (P4,000,000 x 21% x 2.2832) 1,917,888 4,547,888
Difference 452,112 9%

Note: Since the terms are not modified substantially, the note is not considered extinguished.
The difference is recognized in profit or loss in accordance with PFRS 9 par. B5.4.6.

Question No. 35 - D
PVF at 1.5%,
Cash flows 20 periods Present value
Principal 1,000,000 0.7425 742,500
Interest 12,500 17.1686 214,608
Issue price without 957,108

Issue price with 1,045,000


Issue price without (957,108)
Equity component - conversion feature 87,892

Question No. 36 - C
Sales 3,380,000
Cost of sales (3,000,000)
Profit on sale 380,000
Interest income [(P3,380,000 - P500,000) x .1] 288,000
Total income - 2022 668,000
Question No. 37 - B
Projected benefit obligation, Dec. 31 (see below) 10,280,000
Less fair value of plan assets, Dec. 31 (see below) 8,870,000
Accrued (prepaid) benefit expense, Dec. 31 1,410,000

Projected benefit obligation, Jan. 1 9,000,000


Service cost 1,000,000
Page 42 of 52
Interest on projected benefit obligation (P9,000,000 x .1) 900,000
Actuarial loss on projected benefit obligation 160,000
Benefits paid to retirees (780,000)
Projected benefit obligation, Dec. 31 10,280,000

Fair value of plan assets, Jan. 1 8,000,000


Interest on plan assets (P8,000,000 x .1) 800,000
Actuarial gain on plan assets (P850,000 - P800,000) 50,000
Employer contributions 800,000
Benefits paid to retirees (780,000)
Fair value of plan assets, Dec. 31 8,870,000

Alternative computation:
Accrued (prepaid) benefit expense, Jan. 1 (P9,000,000 - P8,000,000) 1,000,000
Defined benefit expense:
Service cost 1,000,000
Interest cost (P1,000,000 x .1) 100,000
Remeasurement - DBO 160,000
Remeasurement - Plan assets [(P8M x .1) - P850,000] (50,000) 1,210,000
Employer contributions (800,000)
Accrued (prepaid) benefit expense, Dec. 31 1,410,000

Question No. 38 - A
Question No. 39 - D
Question No. 40 - C

Question No. 41 - B
Construction costs 200,000,000
Estimated demolition and fuel rod sealing operation 120,000,000
Total cost 320,000,000
Accumulated depreciation, 12/31/22 (P320 million x 1/10) (32,000,000)
Carrying amount, 12/31/22 288,000,000

Question No. 42 - B
CA of asset, end of year (P1,000,000 - P100,000) 900,000
Tax base -
Taxable temporary difference 900,000

Deferred tax liability, end of year 270,000

Question No. 43 - C
Question No. 44 - C
Compensation expense - 2022 (60,000 x P7 x 1/2) 210,000

Question No. 45 - C
Share premium, 12/31/21 (P160,000 + P600,000) 760,000
April 30 - retirement of PS [1,000 x (P25 - P20)] (5,000)
July 31 - re-issuance of TS [800 x (P50 - P42.5*)] 6,000
Total share premium, 12/31/22 761,000

* Cost per share of treasury shares after share split (P85/2)

Question No. 46 - B
Remaining cost of treasury shares (P5,000 x P12) 60,000

Question No. 47 - B
Total equity 7,920,000
Less Preference equity
Liquidation value (20,000 x P110) 2,200,000
Dividends (P2M x .12 x 3) 720,000 2,920,000
Ordinary equity 5,000,000
/Outstanding ordinary shares 200,000
Book value per share - ordinary 25.00

Page 43 of 52
Personal copy of Barbin, Rafael R.
(rafaelbarbin123@gmail.com)
Global Reciprocal Colleges,Inc-Caloocan

Question No. 48 - C
Profit to ordinary for 2021 (1,000,000 x P4.8) 4,800,000
Add preference share dividends in 2021 (P2,000,000 x 10%) 200,000
Profit for 2021 5,000,000
Increase in profit for 2022 (P5,000,000 x 10%) 500,000
Profit for 2022 5,500,000
Less preference share dividends 2022 (P2,000,000 x 1/2 x 10%) 100,000
Profit to ordinary for 2022 5,400,000
Divide by weighted aveg ordinary shares outs. (1 million x 3) 3,000,000
Earnings per share for 2022 1.80

Question No. 49 - B
Question No. 50 - B
Question No. 51 - C
Question No. 52 - B
Question No. 53 - A
Question No. 54 - A
Cash and cash equivalents (P850,000 + P180,000) 1,030,000
Short-term investments (P630,000 - P180,000) 450,000
Accounts receivable (P2,800,000 - P650,000 - P50,000) 2,100,000
Interest receivable 50,000
Prepaid expenses 320,000
Total current assets 3,950,000

Question No. 55 - C
Accounts payable 1,730,000
Accrued expenses 450,000
Notes payable, due on 9/30/22 500,000
Mortgage payable - current portion (P50,000 x 2) 100,000
Total current liabilities 2,780,000

Question No. 56 - B
Sales 2,000,000
Cost of goods sold (P365,000 + P1,555,000 - P15,000 - P325,000) (1,580,000)
Salaries (120,000)
Delivery expense (22,000)
Dividend income 18,000
Income on sales of fixed asset 7,000
Light and power (80,000)
Travel and transportation (18,000)
Interest and bank charges (35,000)
Miscellaneous operating expenses (6,000)
Bad debts (4,000)
Depreciation (15,000)
Income tax (43,500)
Profit 101,500

Question No. 57 - B
Profit 8,000,000
Increase in accounts receivable (1,300,000)
Increase in allowance for doubtful accounts 200,000
Decrease in prepaid insurance 150,000
Increase in accounts payable 300,000
Net cash provided by operating activities 7,350,000

Question No. 58 - C
Minimum external revenue (P120,000,000 x 75%) 90,000,000

Question No. 59 - B
Question No. 60 - C

Page 44 of 52
Personal copy of Barbin, Rafael R.
(rafaelbarbin123@gmail.com)
Global Reciprocal Colleges,Inc-Caloocan
Nominal
peso Conv. factor Constant peso
Net monetary items, 1/1 80,000 1.2100 96,800
Sales 300,000 1.1000 330,000
Purchases (120,000) 1.1000 (132,000)
Operating expenses (90,000) 1.1000 (99,000)
Income tax expense (60,000) 1.1000 (66,000)
Cash dividends declared, 9/1 (20,000) 1.0000 (20,000)
Net monetary items, 12/31 90,000 109,800

Net monetary items, 12/31 - constant peso 109,800


Less net monetary items, 12/31 - nominal peso 90,000
Purchasing power loss (gain) 19,800

Question No. 61 - D
Fixed quarterly rent 100,000
Additional rent
Cumulative additional rent, 6/30:
{[(P6.2M x 2)–P10M] x .05 x 2/4} 60,000
Cumulative additional rent, 3/31 {[(P3.2M x 4)–P10M] x .05 x 1/4} (35,000) 25,000
Warranty expense
Cumulative warranty expense, 6/30 (P6.2M x .1) 620,000
Cumulative warranty expense, 3/31 (P3.2M x .05) (160,000) 460,000
Insurance (P400,000/4) 100,000
Property tax (P600,000/4) 150,000
Repairs 180,000
Loss on inventory write down to NRV (P500,000 - P300,000) 200,000
Total expenses - 2nd quarter 1,215,000

Question No. 62 - D
Shareholders' equity, 12/31 (P18M - P7M) 11,000,000
Shareholders' equity, 1/1 (6,000,000)
Net increase in equity/Profit 5,000,000

Question No. 63 - A
Total assets, end of year 80,000
Less Equity, end of year:
Share capital, end 15,000
Retained earnings, end
Retained earnings, beg. 22,000
Net income 15,000
Dividends declared (7,000) 30,000 45,000
Total liabilities, end of year 35,000

Question No. 64 - C
Policy A (P14,400 x 7/12) 8,400
Policy B -
Policy C (P11,200 x 4/16) 2,800
Prepaid insurance, as adjusted 11,200
Prepaid insurance, as recorded 35,200
Increase (decrease) in Prepaid insurance (24,000)

Question No. 65 - C
Question No. 66 - B
Question No. 67 - C
Question No. 68 - B
Question No. 69 - B
Question No. 70 - C

Page 45 of 52
Personal copy of Barbin, Rafael R. (rafaelbarbin123@gmail.com)
Global Reciprocal Colleges,Inc-Caloocan

Suggested Solution – AFAR

1. B
AA (P50,000 + P40,000) x 60% P 54,000 P&L AA 50%
BB (P50,000 + P40,000) x 40% 36,000 BB 50%
Total P 90,000 TOTAL 100%

2. B
AA BB CC TOTAL
Capital, 01/01/15 P 54,000 P 36,000 P --- P 90,000 P&L AA 40%
1ST QTR LOSS ( 7,500) (7,500) --- (15,000) BB 40%
Capital, 03/31/15 P 46,500 P 28,500 --- P 75,000 CC 20%
Purchase of interest ( 9,300) (5,700) 15,000 --- TOTAL 100%
Capital, 04/01/15 P 37,200 P22,800 P15,000 P 75,000

3. C
AA [ ( P46,500 x 20%) - (P 3,000 x 50%) ] P 7,800
BB [ ( P28,500 x 20%) - (P 3,000 x 50%) 4,200
TOTAL p 12,000

4. C
AA BB CC DD TOTAL P&L AA 28%
Capital, 04/01/15 P37,200 P22,800 P15,000 --- P 75,000 BB 28%
2nd QTR PROFIT 4,000 4,000 2,000 --- 10,000 CC 14%
Capital, 06/30/15 P41,200 P26,800 P17,000 --- P 85,000 DD 30%
DD’s contribution ( 1,800) (1,800) ( 900) 34,500 30,000 TOTAL 100%
Capital, 07/01/15 P39,400 P25,000 P 16,100 P34,500 P115,000

5. A
AA BB CC DD TOTAL
Capital, 07/01/15 P39,400 P25,000 P16,100 P34,500 P115,000
3rd QTR PROFIT 4,200 4,200 2,100 4,500 15,000
Capital, 09/30/15 P43,600 P29,200 P18,200 P39,000 P130,000 RPL AA 28
Cash paid to BB (30,000) (30,000) CC 14
Bonus to BB ( 311) 800 ( 156) ( 333) --- DD 30
Capital, 10/01/15 P43,289 P --- P18,044 P38,667 P100,000 TOTAL 72

6. A
AA CC DD TOTAL
Capital, 10/01/15 P 43,289 P 18,044 P 38,667 P100,000
4th QTR PROFIT 7,778 3,889 8,333 20,000
BBL P 51,067 P 21,933 P 47,000 P120,000
Realization loss/
Liquidation expenses (4,667) (2,333) ( 5,000) (12,000)
Cash paid partners P46,400 P19,600 P42,000 P108,000

7. C
8. A
Gross loss (P1,695,000 + P225,000) P1,920,000
Less: (Estimated gain, P945,000 + Contingent assets, P750,000) 1,695,000
Net Loss P 225,000
Less Book Value of SHE (P1,500,000 – P600,000) 900,000
Amount due to stockholders upon liquidation P 675,000
Pro-rata payments to stockholders (P675,000/P900,000) P0.75 to P1.00

Page 46 of 52
Personal copy of Barbin, Rafael R. (rafaelbarbin123@gmail.com)
Global Reciprocal Colleges,Inc-Caloocan

9. A
Total estimated cash (P28,000 + P72,000 + P61,600 + P26,400) P 188,000
Less priority claims: Income taxes P 6,400
Note payable (secured portion) 72,000
Salaries payable 4,800
Administrative/liquidation expenses 16,000 99,200
Estimated net amount available for non-priority claims P 88,800

10. C
Net free assets (please see above computation) P 88,800
Less non-priority claims
Notes payable (unsecured portion) P 24,000
Accounts payable 68,000
Bonds payable 56,000 148,000
Estimated deficiency P(59,200)
Estimated recovery rate (ERR): P88,800 / P148,000 60%

Notes payable : Total Book Value P96,000


Secured portion 72,000 x 100% P 72,000
Unsecured portion P 24,000 x 60% 14,400
P 86,400
11. B
RBNI (P 48,000 – P8,800 – P24,000 – P15,360 – P9,600 + P16,000) P6,240
Add allowance realized from branch sales to outsiders:
Allowance before adjustment (AFOVOBI) P 5,760
Required allowance on the BEI (P13,440/120%) x 20% 2,240 3,520
True branch net income P9,760
12. A
[P162,000 + P202,500)/135%] x 35% P 94,500

13. A

Adjusted reciprocal balance (380,625 + 6,750) P 387,375


Add other adjustments (4,375 + 1,125) 5,500
Unadjusted balance of Home Office account P 392,875

14. A
COS: Beg 38,400
Shipments 99,840
End (30,720)
COS at BP 107,520
Divide by 1.2
COS at Cost 89,600
Multiply by 0.2
AFOVBI (COS) 17,920

15. A
Br: Sales 134,400
Less: COS at Cost 89,600
GP 44,800
Less:OPEX 23,040
Net Income 21,760

Page 47 of 52
Personal copy of Barbin, Rafael R. (rafaelbarbin123@gmail.com)
Global Reciprocal Colleges,Inc-Caloocan

16. C
HO: Sales 384,000
Less: COS
Beg 76,800
Purchases 320,000
Shpmnts (83,200)
End (62,720) 250,880
GP 133,120
Less:OPEX 46,080
Net Income 87,040
Br True Net Income 21,760
CNI 108,800
17. D
Cost of investment P 385,000
Less book value of net assets 437,500
Overstatement of building P (52,500)
Annual amortization P 5,250
CNI = P78,750 – P8,750 + P5,250) P 75,250

18. D
Parent’s Retained Earnings, 1/2/15 P 446,250
Plus CNI (subsidiary is fully-owned) 75,250
Less Parent’s Cash dividends 8,750
Consolidated Retained Earnings, 12/31/15 P 512,750

19. C
20. B
21. D
CNI = [(P122,500 – P42,000) + P78,750] + P1,531 –
P 1,313 + P4,725 – P3,150 P 161,043

22. B
PNI (from own operations (P122,500 – P42,000) P 80,500
Plus parent’s investment income:
Adjusted SNI (P78,750 + P1,531 – P1,313) P78,968
X CI% 80%
Investment income before dwnstrm adjstmnt P63,174
Add net dwnstrm adjstmnt (P4,725 – P3,150) 1,575 64,749
Share of parent’s shareholders from CNI P 145,249

23. B
Adjusted SNI P 78,968
X by NCI% 20%
NCI in CNI (P161,043 – P145,249) P 15,794
24. A
Decrease in spot exchange rate (P0.4295 – P0.4245) P 0.005
Multiply by FC 100,000
Foreign exchange gain P 500

25. A
Amount Due from Broker (FC 100,000 x P0.4300) P 43,000

26. D
Increase in spot exchange rate (P0.4345 – P0.4245) P 0.01
Multiply by FC 100,000
Foreign exchange loss P ( 1,000)

Page 48 of 52
Personal copy of Barbin, Rafael R. (rafaelbarbin123@gmail.com)
Global Reciprocal Colleges,Inc-Caloocan

27. D
Decrease in Forward exchange rate (P0.4300 – P0.4325) P0.0025
Multiply by FC 100,000
Foreign exchange loss P ( 250)

28. C
Overall loss on FCT (P0.4345 – P0.4295) x FC100,000 P ( 500)
Overall gain on FEC (P0.4345 – P0.4325) x FC100,000 200
Net overall loss, also (P0.4325 – P0.4295) x FC100,000 P ( 300)
29. C
Percentage (%) of Completion Method
CIP Bohol (P254,167 - P35,417) P218,750
Davao (P100,000 + P 25,000) 125,000 P343,750
PB Bohol + Davao (P180,000 + P150,000) 330,000
Contract Asset P 13,750

30. A
Zero Profit Method
CIP Bohol P218,750
Davao 100,000 P318,750
PB 330,000
Contract Liability P(11,250)

31. A
Cash Collection Actual cost to date (2022)
PB P1,500,000 CIP P1,600,000
AR, end ( 500,00 ) Gross profit in 2022 200,000
Collection P1,000,000 Cost incurred in 2022 P1,400,000

32. D
33. A
Total assets (P12,500 + P17,500 + P23,000 +
78,000 + P5,500 + P12,500) P 149,000
34. B
Consolidated net income (P820,000 – P525,000 –
212,500 – P2,100) P 80,400
35. A
36. D
37. B
Cost P1,400,000
Less: FV of net assets (P36,000 + P457,000 + P133,000 +
P900,000 – P350,800) 1,175,200
Goodwill P 224,800
38. D
Materials Conversion
C&T 1,900 1,900 1,900
Loss-Abnormal 800 800 240
2,700 2,700 2,140
39. D
Units Sales Value ESTIMATED COST Unit
Prod Prod @ Final Point To complete To sell NRV Joint Cost Total Cost Cost
X 800 P 40,000 P 5,000 P 4,000 31,000 21,700 26,700 33.38
A 600 34,200 4,000 4,200 26,000 18,200 22,200 37.00
K 400 32,000 3,000 2,000 27,000 18,900 21,900 54.75
1,800 P106,200 P12,000 P10,200 84,000 58,800 70,800 -
-
Page 49 of 52
Personal copy of Barbin, Rafael R. (rafaelbarbin123@gmail.com)
Global Reciprocal Colleges,Inc-Caloocan

40. B (Refer to above solution)


41. C
PROD I PROD II SERVICE A SERVICE B
Budgeted costs P 480,000 P 520,000 P40,000 P60,000
Allocation of A’s cost 19,630 18,518 (40,000) 1,852
Allocation of B’s cost 31,827 30,025 -- (61,852)
Totals P531,457 P 568,543 -- --

42. A (Refer to above solution)


43. A (Refer to above solution)
44. C
Job #901
FO Applied
Set-up P 800
Inspection 440
Materials handling 300
Engineering 1,250
2,790

45. C
46. B
Materials Conversion
C&T 38,000 38,000 38,000
IP, end 11,000 5,500
Loss- Normal 760 760
Loss-Abnormal 240 240
50,000 38,000 44,500
Unit costs:
Preceding dept (P38,000 + P140,000)/50,000 P 3.56
Materials (P21,200 + P70,000)/38,000 2.40
Conversion (P81,300 + P292,500)/44,500 8.40
Cumulative P 14.36

Finished goods: (P38,000 x P14.36) P545,680


Plus CNLU (760 X P3.56) + (760 x P8.40) 9,090
Total cost P554,770

47. B
FOC (240 x P3.56) + (240 x P8.40) P 2,870

48. A
Cost from PD (11,000 x P3.56) + (5,500 x P8.40) P 85,360

49. A

50. D
Total cost (P532,000 + P8,218) - (P15,960 x 80%) P 527,450
Divide by units produced (400 – 15) 385
Unit cost P 1,370

51. D
Total cost [P532,000 - (15 x P1,330)] P 512,050
Divide by units produced (400 – 15) 385
Unit cost P 1,330

52. A
53. A
Page 50 of 52
Personal copy of Barbin, Rafael R. (rafaelbarbin123@gmail.com)
Global Reciprocal Colleges,Inc-Caloocan

54. D
55. B
Operators’ equal share over the depreciated value of the fixed asset
at 12/31/22 (P200,000 x 90%) x 1/3 P 60,000
Less unamortized deferred gain (CC only) (P15,000/3) x 90% 4,500
Amount of machinery in CC’s balance sheet, 12/31/22 P 55,500
The same amount is derived by using the cost (P185,000 x 90%) x 1/3 P55,500.

56. B
57. A
Sales (Purch 100,000 + Expenses 13,000 + Cr Bal + 80,000) 193,000
COS: Beg 100,000
End ( 1,200 + 1,500) (2,700) 97,300
Gross Profit 95,700
Expenses (6,000+7,000) 13,000
Net Income 82,700

TANTE LEN
Investment 50,000 50,000
Share in NI (82,700/2) 41,350 41,350
Unsold Merch (1,200) (1,500)
Cash Due 90,150 89,850

COH
Sales (50,000+ 6,000+42,000) 98,000
(50,000 + 7,000+38,000) 95,000
Expenses (6,000) (7,000)
92,000 88,000

Cash Settlement
(90,150- 92,000) (1,850)
(89,850- 88,000) 1,850

58. D ( see previous solution)


59. C
Contract Price:
DP 100,000
Installment (50,000 * 3.17)158,500
258,500
Less: Direct and Indirect cost 42,000
Add: Interest Income (158,500 * 10%) 15,850
Profit 232,350

60. A
Payment Interest Payment to principal Balance
Beg 158,500
12/31/21 50,000 15,850 34,150 124,350
13/31/22 50,000 12,435 37,565 86,785

61. A

Revenue 258,500 /4 64,625


Interest revenue 15,850
Less: Direct Cost 27,000/4 6,750
Indirect Cost 15,000
Profit 58,725

62. A
Revenue 258,500 /4 64,625
Interest revenue 12,435
Less: Direct Cost 27,000/4 6,750
Profit 70,310

Page 51 of 52
Personal copy of Barbin, Rafael R. (rafaelbarbin123@gmail.com)
Global Reciprocal Colleges,Inc-Caloocan

63. D
64. D
65. C
66. C
67. A
68. C
69. B
70. D

END

Thank you for participating in Team PRTC Nationwide Online Open Pre-board Examination for
October 2022 Batch

Page 52 of 52

You might also like